Ichigo USLME Med Management P3

1) A 46-year-old female is brought to the emergency department because of visual blurring and altered mental status. She was recently treated for gout. Physical examination shows moderate to severe splenomegaly and positive sternal tenderness. Laboratory studies reveal a WBC count of 120, 000/uL with left shifted myeloid series and basophilia. Bone marrow exam shows 3% blasts. Repeat CBC is ordered. The hematologist-on-call decides to start leukophereses emergently. What another drug should be given to this patient?
. Cladribine
. Hydroxyurea
. Cyclophosphamide
. Interferon alpha
. Imatinib mesylate
2) A 64-year-old male patient with deep vein thrombosis is being treated with unfractionated heparin. On the 4th day of treatment, his platelet count drops to 80,000/cmm. His previous platelet count on day 2 was within normal range. He denies any bleeding-related complications, and is completely asymptomatic. His complete examination does not reveal any signs of bleeding. His blood pressure is 128/80mm of Hg, pulse is 78/min, and respirations are 20/min. He is afebrile. What is the most appropriate next step in the management of this patient?
. Switch to low molecular weight heparin
. Stop heparin and start warfarin
. Stop heparin
. Start plasmapheresis
. Stop heparin and start platelet transfusion
3) A 19-year-old man comes to the office and says, "Doctor! I have been having a peculiar problem lately. My stool has a funny color. First, it was black and almost sticky. Yesterday, it became maroon. In fact, today I saw some bright red blood." He denies any associated pain or fever. He is adopted, and his family history is unknown. His vital signs are stable. Physical examination is normal, but his stool is positive for occult blood. Laboratory studies reveal a hematocrit of 29% and hemoglobin concentration of 9.6 g/dL. Colonoscopy reveals hundreds of colonic polyps, which are identified as adenomatous polyps with the biopsy. What is the appropriate recommendation for this patient at this point?
. Reassure the patient as the polyps are most probably benign and have no long-term complications
. Perform regular colonoscopy and biopsy every three years from now on to check for any malignant change
. Start regular colonoscopy and biopsy eight years from now
. The patient needs elective procto-colectomy
. The patient needs close surveillance with regular F OBT and CEA levels
4) A 6-year-old, African-American boy presents with fever and pain in his extremities for the last several hours. The pain is unrelated to movement or posture. His pulse is 102/min, blood pressure is 110/70 mm Hg, temperature is 38°C (100.4°F), and respirations are 18/min. Physical examination reveals pallor, jaundice, and splenomegaly. Both lower legs are tender to palpation. Lab tests show the following: Hematocrit 20%, WBC count 13,000/micro-L, Platelet count 180,000/miro-L, Bilirubin 5.0 mg/dL, Direct bilirubin 1.2 mg/dL, Reticulocyte count 11%. Peripheral blood smear shows sickle shaped cells. Hemoglobin electrophoresis confirms the diagnosis of sickle cell anemia. After treating the child with analgesia, hydration, and oxygen therapy, the painful episode subsides. Which of the following measures should be employed to prevent aplastic crisis in this child?
. Supplementation with folic acid
. Supplementation with iron
. Vaccination against parvovirus
. Vaccination against pneumococcus
. Treatment with hydroxyurea
5) A 22-year-old white male presented to the emergency room (ER) with the sudden onset of acute right upper quadrant pain. The ultrasound showed cholelithiasis. Initial evaluation revealed hemoglobin of 9 gm/dl with an MCV of 90 fl and a total reticulocyte count of 1000 cells per microliter. Peripheral smear revealed polychromatophilia and spherocytes. Liver function tests revealed an elevated indirect bilirubin and normal hepatic enzyme levels. Physical examination is consistent with pallor. The patient's parents were killed in an accident when he was 8-year-old, and the patient does not know anything about the family history. Which of the following is the most correct statement about this patient's condition?
. His condition is classically transmitted as autosomal recessive disorder
. He will probably depend on transfusions
. This patient should be placed on folic acid supplementation
. Vaccination against parvovirus has shown to decrease morbidity and mortality
. This patient's mean corpuscular hemoglobin concentration (MCHC) is likely to be very low
6) A 70-year-old man presents to the urgent care clinic complaining of extensive bruising and bleeding gums. His medical history is significant for atrial fibrillation for which he takes metoprolol and warfarin. He has been stable on his current dose of warfarin for several months. He denies any recent use of NSAIDs or other over-the-counter medications. The patient reports that he has been taking his warfarin as prescribed and has been trying to "live a healthier life" lately. On further questioning, he reports trying to exercise more often, taking numerous vitamin supplements daily, and eating more vegetables. His new diet consists mainly of vegetables like spinach, Brussels sprouts, and broccoli, as well as copious amounts of green tea. Which of the following lifestyle changes is most likely responsible for this patient's bleeding?
. Brussels sprouts
. Green tea
. Spinach
. Increased exercise
. Vitamin supplements
7) A 26-year-old man comes with his girlfriend to the emergency department due to a very high fever. He just finished his second cycle of BEP (bleomycin, etoposide, cisplatin) chemotherapy for metastatic seminoma 4 days ago. Other than his fever, he has no complaints. He denies any chest pain, cough, diarrhea or any rash. He stopped smoking ever since he was diagnosed with his "deadly disease," but drinks alcohol occasionally. His temperature is 38.9°C (102°F), blood pressure is 118/70 mm Hg, pulse is 102/min, and respirations are 19/min. Physical examination reveals a pale man without any eyebrows or eyelashes. Chest auscultation is clear. Blood tests reveal: WBC 690/mm3 with 9% neutrophils, Hemoglobin 8.6 g/dl, Hematocrit 25%, Platelets 74,000/mm3. What is the best next step in the management of this patient?
. Give acetaminophen and send him home
. Obtain blood cultures and give cefepime
. Obtain blood cultures and give vancomycin
. Order blood cultures and wait for the results
. Give blood, platelet, and G-CSF transfusion
8) A 34-year-old woman comes to the ER because of right lower leg swelling, redness, and pain. She has no significant past medical history and does not use any medications. Her mother has a history of pulmonary embolism. Her temperature is 36.7°C (98°F), pulse is 80/min, respirations are 16/min, and blood pressure is 120/76 mmHg. Examination shows tenderness to palpation in the right calf. Compressive ultrasonogram shows a deep vein thrombosis of the right leg. Further evaluation reveals an elevated plasma homocysteine level. She is started on heparin and warfarin therapy. What other additional therapy is indicated in this patient?
. Clopidogrel
. Streptokinase
. Vitamin E
. Pyridoxine
. Simvastatin
9) A 55-year-old male comes to the physician's office because of fatigue. He denies any other symptoms. His vital signs are stable. Examination shows pallor, massive splenomegaly, and mild hepatomegaly. CBC reveals pancytopenia with striking monocytopenia. His peripheral blood smear is shown below. Bone marrow biopsy shows a dry tap. What is the most appropriate treatment for this patient's condition?
. Bone marrow transplantation
. Cladribine
. Cyclophosphamide
. CHOP regimen
. Chlorambucil and prednisone
10) A 36-year-old male comes to your office complaining of progressive fatigability. His past medical history is significant for infiltrative pulmonary tuberculosis diagnosed two months ago. His current treatment includes isoniazid, rifampin, ethambutol, and pyrazinamide. Laboratory values are: Hemoglobin 8 g/dl, MCV 77 fl, MCHC 30%, ESR 17 mm/hr, Serum iron 170 micro-g/dl (N 50- 150 micro-g/dl ), Total iron binding capacity (TIBC) 280 micro-g/dl (N 300-360 micro-g/dl). Microscopy reveals two populations of red blood cells (RBC)-hypochromic and normochromic. What is the next best step in the management of this patient?
. Bone marrow biopsy
. Iron preparations
. Folic acid
. Folic acid and vitamin B12
. Pyridoxine
11) A 56-year-old male is admitted to the ICU for the management of hepatic encephalopathy. He is a known alcoholic. He is started on oral neomycin, lactulose, and a low-protein, high-carbohydrate diet. His blood pressure is 116/78 mm Hg, pulse is 78/min, temperature is 37.5°C (99.5°F), and respirations are 18/min. His labs reveal the following: Hb 10.2 g/dl, WBC 7,400/cmm, Hct 27%, Platelets 4 2,000/mm3, PT 18 sec, INR 2.02, PTT 31 sec. He has no evidence of active bleeding. Which of the following should be administered to this patient at this time?
. Fresh frozen plasma
. Cryoprecipitate
. Vitamin K
. Platelets
. Packed RBC
12) A 32-year-old Italian-American man presents to your office for a routine check-up. He works as a business executive and admits to being under a lot of stress recently. He drinks alcohol occasionally and smokes one pack of cigarettes per day. Laboratory analyses reveal: Hemoglobin 10.1 mg/dl, RBC count 4.0 x 10^12/L, MCV 70fl, WBC count 5,500/mm3, Platelets 170,000/mm3, Serum calcium 10.1 mg/dl, Serum potassium 4.5 meq/L, Serum sodium 135 meq/L. Serial fecal occult blood tests are negative. A peripheral blood smear reveals the following: Which of the following is the best treatment for this patient?
. Iron
. Folic acid
. Cobalamin
. Reassurance
. Erythropoietin
13) A 47-year-old man comes to the office for the evaluation of a sore over his right ear. His sore bleeds and oozes, and has remained open for 3 weeks. He thinks that it has been growing slowly over the past year. He works for the postal service (USPS), and is usually out all day delivering mail. He admits to smoking approximately 2 packs of cigarettes daily for the last 13 years. Physical examination reveals a punched out 1.5 cm ulcer on his right auricle. There are no enlarged lymph nodes. Full thickness biopsy reveals proliferating malignant cells which arise from the epidermis and extend into the dermis in lobules or strands. What is the best next step in the management of this patient?
. Surgical excision with 1-2 mm of clear margins
. Wide excision with at least 5 em margins
. Scraping
. Observe for 3 weeks and then examine
. Local chemotherapy with anticancer creams
14) A 56-year-old man complains of fatigue and occasional palpitations. He has a 20-year history of diabetes mellitus and takes daily insulin therapy. He receives hemodialysis three times a week for end-stage renal disease. He was recently started on erythropoietin therapy for anemia (his pretreatment hemoglobin was 8.0 mg/dl). Physical examination at this visit reveals pale conjunctiva. Repeat laboratory studies show: Hemoglobin 9.2 mg/dl, MCV 77 fl, MCHC 30 g/dl, WBC count 7,000/mm3, Platelets 150,000/mm3, Hemoglobin A1c 7.5%, ESR 15 mm/hr. Which of the following is likely to be helpful in improving this patient's symptoms?
. Tighter blood glucose control
. Higher erythropoietin dose
. Iron supplementation
. Folic acid supplementation
. Splenectomy
15) A 48-year-old Caucasian female is undergoing a routine work-up 9 months after radical mastectomy due to right-sided breast cancer. She has no present complaints, and says that she is 'full of energy'. Chest radiograph reveals an isolated pulmonary nodule, 2.0 cm in diameter, in the middle lobe of the right lung. CT confirms that the lesion is solitary with poorly defined margins, but completely within lung parenchyma. Bone scan is negative. CT abdomen and pelvis is normal. Transthoracic biopsy demonstrates adenocarcinoma with positive estrogen receptors (ER) and no HER-2/neu overexpression. Which of the following is the best management strategy for this patient?
. Surgery
. Systemic chemotherapy
. Hormone therapy
. Pretreatment with monoclonal antibodies against HER-2/neu
. Combined chemotherapy and endocrine therapy
16) A 56-year-old white male is being evaluated for having isolated IgG elevation. The level of serum IgG is 2400 (Normal is 700 to 1500), while the levels of the other paraproteins are within normal limits. The patient was previously admitted with a diagnosis of pneumonia, which resolved completely with levofloxacin. His other laboratory test results are as follows: Hemoglobin 13.0 g/dl, Serum Ca 8.0 mg/dl, BUN 18 mg/dl, Creatinine 1.1 mg/dl. Bone marrow biopsy reveals 2% of plasma lesions. What is the most appropriate next step in the management of this patient?
. Reassurance and regular follow-up
. Melphalan and prednisolone
. Interferon
. Combination chemotherapy
. Autologus bone marrow transplantation
17) A 55-year-old Caucasian male is hospitalized after a massive hemorrhage from a duodenal ulcer. The hemorrhage was stopped with endoscopic manipulation. Two litters of Ringer lactate were infused and two units of packed red blood cells (RBC) ordered. Ten minutes after the transfusion of packed RBC had been started; the patient was complaining of tremor and feeling 'chilly.' His temperature is 39.4° C (103° F), blood pressure is 120/76 mmHg, pulse is 100/min, and respirations are 24/min. The transfusion was stopped, and acetaminophen was given. Direct antiglobulin test and plasma free hemoglobin level are negative. Urinalysis is normal. The reaction abated three hours after the transfusion had started. Which of the following actions could have prevented the reaction described?
. Warming the blood
. Using whole blood
. Careful cross matching of the blood
. Infusing calcium gluconate
. Washing cells
18) A 44-year-old white male comes to the office and says, "I have had this persistent, upper abdominal pain for the past 2 months. My wife has peptic ulcer, and I think I also have an ulcer down there, so I take some of her medicines. At first, the medicines made the pain go away, but soon after the pain proved to be too tough for the medicines. I hardly want to eat anything now. Do you think all this is due to some reaction to those medicines?" Physical examination reveals mild tenderness in the epigastric region. Esophagogastroduodenoscopy reveals a mass in the pylorus, which the biopsy identifies as an infiltrating, low-grade, gastric mucosa-associated lymphoid tissue (MALT) lymphoma. Further work-up concludes that there are no regional lymph nodes involved. What is the best next step in the management of this patient?
. Radical gastrectomy with en bloc resection of spleen and regional nodes
. Give a combination of omeprazole, clarithromycin, and amoxicillin
. Give combination chemotherapy
. Radiotherapy
. Observation and strict surveillance
19) A 42-year-old African-American woman presents to your office with bilateral knee pain that severely limits her mobility. She also complains of joint stiffness in the morning that takes several hours to improve. Her hand joints are symmetrically swollen. She has been taking over-the-counter ibuprofen and aspirin with little relief of symptoms. She has no other medical problems and does not use tobacco, alcohol, or illicit drugs. Her vital signs are within normal limits. Physical examination shows pain and swelling of both wrists and knees. Laboratory studies reveal: Hemoglobin 8.4 mg/dl, Serum iron 30 μg/dL (normal 50-150 μg/dL), Total iron binding capacity 230 μg/dL (normal 300-360 μg/dL), Ferritin 300 ng/dl. Which of the following is most likely to improve this patient's anemia?
. Iron supplementation
. Folic acid supplementation
. Cyanocobalamin supplementation
. Infliximab
. Splenectomy
20) A 25-year-old, G3P0 white female in her 8th week of gestation comes to the antenatal clinic. Her obstetric history is significant for two second trimester abortions. She is currently taking folic acid and iron supplementation. She does not smoke, but she drinks alcohol on social occasions. She is in a monogamous relationship with her husband. She has never been diagnosed with or treated for any STDs. Laboratory studies reveal the following: VDRL positive, FTA-ABS negative, Hct 33%, WBC 7,000/mico-L, Platelets 70,000/micro-L, PT 10 sec, APTT 40 sec. What is the most appropriate next step in the management of this patient?
. Give heparin and aspirin
. Repeat FTA-ABS
. Repeat VDRL
. Single dose of benzathine penicillin
. Three doses of benzathine penicillin
21) A 64-year-old male, who rarely sees the doctor and doesn't believe in screening procedures, comes to office with the complaint of severe back pain. He has not felt comfortable for the past 4 months and is having severe pain in the back. He has found it difficult to sleep. He has hypertension, for which he takes hydrochlorothiazides regularly. He thinks that he has lost some weight. On examination, his BP is 165/90mm of Hg, while rest of vitals are stable. On rectal examination you find a rock hard 2 cm nodule in prostate. Biopsy shows poorly differentiated adenocarcinoma. Bone scans shows wide spread metastasis all over his skeleton. What is the most appropriate treatment for his metastatic disease?
. Leuprolide
. Diethylstilbestrol (DES)
. Flutamide
. Radicle prostatectomy
. Complete androgen blockade with Leuprolide and Flutamide
22) A 65-year-old woman is very worried because she recently had a close family member and friend die, one after the other. First, her 85-year-old mother died of an ovarian cancer that was diagnosed 3 years ago. Last week, she lost her best friend to lung cancer that metastasized to the liver and brain. Today, her 58-year-old sister learned that she has breast cancer. She has regular pap smears and breast examinations. She has read in the paper that there are a few ways that have proved to be very successful in preventing breast cancer. Which of the following measures has the best evidence of preventing breast cancer in high-risk women?
. Prophylactic oophorectomy
. BRCA screening
. Low-fat diet
. Tamoxifen
. Having mammograms every month
23) A 16 year-old-girl with hereditary spherocytosis is scheduled for a splenectomy. She was previously managed with folate therapy and occasional blood transfusions, but her anemia became refractory to medical management alone. Before the operation, she is told that she will have an enhanced risk of developing pneumococcal sepsis. She then asks, "How long will these risks last?" What is the best response to her question?
. 2 weeks
. Up to 6 months
. Up to 2 years
. Up to 10 years
. More than 10 years
24) A 42-year-old mildly overweight Caucasian male is being worked-up for his second episode of deep venous thrombosis in two years. Both episodes seem to have been unprovoked. He denies any recent prolonged immobility, long-distance travel or lower extremity trauma. He has no past medical history of diabetes, cancer or liver disease. A thrombophilia work-up is ordered for this patient. Use of which of the following drugs is most likely to give a false-positive result for protein S deficiency?
. Heparin
. Warfarin
. Aspirin
. Clopidogrel
. Simvastatin
25) A 33-year-old computer programmer was diagnosed with ulcerative colitis last year. He had a colonoscopy and biopsy that showed pancolitis, after which no further work-up was done. This is his first visit to a physician in 6 months. He takes aminosalicylates and is in remission, without any symptoms. He is a member of the Ulcerative Colitis Support Group, which recently sent him a letter that said he should be screened for colon cancer. His vital signs are stable. Physical examination reveals mild tenderness on deep palpation of the lower abdomen. What is the best advice for him regarding his concerns?
. He has an increased risk of colon cancer and he should have prophylactic colectomy
. He has an increased risk of colon cancer and he should have colonoscopy every year
. He has an increased risk of colon cancer and he should have periodic colonoscopy eight years later
. Since he does not have any signs and symptoms, he has no increased risk and doesn't need any surveillance
. He has an increased risk of colon cancer, and he should have a biopsy
26) A 72-year-old Hispanic man comes to the clinic with complains of mild headache, and lethargy for the past several days. He complains of cough for the past 12 years, the cough has been bothering him more lately. The cough is mucoid, he has noticed blood in it once in a while. He has smoked 1 pack/day for 29 years. His vitals are, Temperature 37.0°C (98.6°F); BP 120/84 mm Hg; PR 78/min; RR 24/min. On examination of lungs, adventitious sounds are heard over all the lobes and scant crackles at the base. Laboratory studies show: WBC 7,600/mm3, Hemoglobin 13.6, Hematocrit 40%, Platelets 214,000mm3, Sodium 130 mEq/L, Potassium 3.6 mEq/L, Bicarbonate 18 mEq/L, Blood urea nitrogen 16 mg/dl, Creatinine 0.6 mg/dl, Glucose 95 mg/dl, Serum osmolality 260 mOsm/kg (normal 280), Urine osmolality 310 mOsm/kg. A chest x-ray shows a 2-centimeter left upper lobe mass and mediastinal adenopathy. What will be the next best step in management of this patient?
. Prescribe demeclocycline
. Restrict the patient water intake
. Start an infusion of hypertonic saline
. Start furosemide
. A CT of head
27) You are called by the hospice nurse to evaluate a 74-year-old man who refuses to eat. He has terminal colon cancer, which was diagnosed 8 months ago and has metastasized to his liver. He has already gone through chemotherapy with 5-Fiourouracil + oxaliplatin. During the past 4 months, his condition has gotten extremely worse. He has lost 501bs (27.6kg), and currently appears totally wasted. As you approach him, he says, "I don't feel like eating anything, and I don't want any more procedures, but I'll drink any pill that can make me feel better." His abdomen is distended with positive fluid thrills. His liver has several palpable nodules. What is the best pharmacological agent of choice for this patient's anorexia?
. Megestrol acetate
. Dexamethasone
. Dimenhydrinate
. Ondansetron
. Metoclopramide
28) A 67 -year-old man presents to your office with a one month history of progressive fatigue and exertional dyspnea. He has no significant past medical history. Physical examination reveals enlarged, non-tender axillary lymph nodes bilaterally. His abdomen is soft and non-tender and the liver span is 10 cm. Serial fecal occult blood tests are negative. Laboratory values are as follows: Hemoglobin 7.8 mg/dl, MCV 90 fl, MCHC 33 g/dl, Reticulocytes 7%, WBC count 22,000/mm3, Platelets 200,000/mm3. Which of the following is most likely to improve this patient's symptoms?
. Folic acid
. Iron
. Prednisone
. Pyridoxine
. Erythropoietin
29) A 22-year-old woman comes to the office for the evaluation of a breast mass which she discovered while taking a shower 2 months ago. She experiences severe pain in this mass during her menses. She looks very anxious because her 45-year-old friend was diagnosed with breast cancer last year, and who "now has all sorts of medicines that have made her lose her hair." She has no other problems. She has never been pregnant. She is an occasional smoker, and drinks 3-4 beers a week. There is no family history of breast cancer. Her vital signs are stable. Physical examination reveals a 4 x 5 x 6 cm firm, moveable, rubbery mass in her left breast. Ultrasound shows a cystic mass. Needle aspiration yields clear fluid, after which the mass disappears. What is the best approach in the management of this patient?
. Send the fluid for cytology
. Perform a core biopsy
. Observe for 4 weeks
. Order a mammogram to look for other lesions
. Breath CT scan
30) A 28-year-old male comes for complete physical examination for a new job he is starting next week. He is healthy and has no complaints. He smokes 1 pack/day cigarettes for the last 10 years. He drinks 4 oz of alcohol daily. His mother has hypertension and father has heart problems. Vitals signs are with in normal limits. Physical examination is within normal limits except for a painless, hard mass in left testicle. Ultrasound shows high likelihood of a testicular tumor. What is the next best step in management?
. Observe and recheck the mass after one month
. Platinum based chemotherapy
. Trans-scrotal biopsy
. Fine needle aspiration cytology
. Radical orchiectomy
31) A 41-year-old woman comes to the office for the evaluation of a lump on her left breast. She feels, "generally well," and denies having any medical problems. She takes a multivitamin and Tums daily. She stopped smoking after getting married 21 years ago. She drinks wine only on weekends. Her only surgery was a bilateral tubal ligation last year, after giving birth to her fourth child. Physical examination of the left breast reveals a 4 cm lump at the 11 O'clock position and two axillary nodes on the left side. Mammography shows irregular micro-calcifications. The result of the FNA is suggestive for infiltrating ductal carcinoma. Which of the following is the single most important prognostic factor in this patient?
. TNM staging
. Estrogen and progesterone receptor status
. Her-2/neu oncogene expression
. Histological grade of the tumor
. Irregular micro calcifications in mammogram
32) A 29-year-old woman comes to the office of a primary care physician complaining of pain in her right knee for the last 3 months. The pain is getting progressively worse and is not moving anywhere. She has tried ibuprofen but found little comfort. She is a housewife and unable to do household work because of this pain. She does not smoke nor drink and has hypertension well controlled by hydrocholorothiazide. Her vitals are stable and she is afebrile. On examination her right knee is a little swollen and tender with reduced range of motion. On x-ray, there is soap-bubble appearance in the epiphysis of distal femur. What is the next best step in the management of this patient?
. Refer to an orthopedic surgeon
. Perform biopsy in the office
. Order bone scan
. Start antibiotics
. Do rheumatoid factor and anti-nuclear antibody
33) A 55-year-old man has undergone renal transplantation due to end-stage renal failure. His postoperative course was uncomplicated. He is currently taking prednisone and cyclosporine. He is afebrile and his pulse is 80/min, respirations are 14/min, and blood pressure is 130/65 mm Hg. Physical examination is unremarkable. Which of the following should be added to his current medication regimen to prevent opportunistic infections?
. Trimethoprim-sulfamethoxazole
. Oseltamivir
. Itraconazole
. Azithromycin
. Penicillin
34) A 24-year-old man is found to be HIV positive. He is asymptomatic. Physical examination is unremarkable. Laboratory tests show: CD4 count 400/micro-L, HIV viral load 9,000 copies/mL, VDRL negative, Toxoplasma serology negative, PPD test 6mm induration. His chest x-ray is unremarkable. What is the most appropriate next step in the management of this patient?
. Reassurance and repeat testing in 2 months
. Isoniazid and pyridoxine for 9 months
. Rifampin for 9 months
. Rifampin, isoniazid, pyrazinamide, and ethambutol for 2 months, then isoniazid and rifampin for 4 months
. Rifampin, isoniazid, and pyrazinamide for 6 months
35) A very worried 25-year-old woman comes to the office, presents a positive pregnancy test, and says, "I'm six weeks pregnant. Two months ago, I received the rubella vaccine and my doctor told me to avoid getting pregnant for the next three months. I'm very concerned about the health of my unborn baby. I'd like to know all the available options to prevent any harm to my baby." What is the most appropriate next step in the management of this patient?
. Reassurance
. Explain the risks and benefits of abortion
. Advise abortion
. Serological testing for rubella
. Ultrasonography
36) A 30-year-old male has recently been diagnosed with HIV infection. He denies drug abuse. He is currently asymptomatic, and physical examination is unremarkable. He is in a stable heterosexual relationship. Laboratory studies show a CD4 count of 350/microL, HIV viral load of 15,000 copies/mL, negative VDRL, negative toxoplasma serology, tuberculin skin test of 7 mm induration, negative HBsAg, and positive anti-HBsAg antibodies. Hepatitis C antibodies are negative. Complete blood count, serum chemistries, and liver function tests are within normal limits. He has not received any vaccinations since being diagnosed with HIV. Which of the following vaccines is indicated in this patient?
. BCG vaccine
. Hepatitis A vaccine
. Hepatitis B vaccine
. Pneumococcal vaccine
. Meningococcal vaccine
37) A 42-year-old man with advanced HIV infection has a two-week history of pain and difficulty with swallowing. He was given fluconazole for these symptoms one week ago, but his pain has worsened. His current medications include tenofovir, emtricitabine, efavirenz, and trimethoprim-sulfamethoxazole. His CD4 count is 90/microl and viral load is 300,000copies/ml. Endoscopy reveals large, irregular, linear ulcers in the esophagus. A biopsy of the abnormal mucosa reveals tissue destruction and the presence of intranuclear and intracytoplasmic inclusions. Which of the following is the most appropriate pharmacotherapy for this patient?
. Prednisone
. Acyclovir
. Pentamidine
. Ganciclovir
. Itraconazole
38) A 26-year-old man presents to the physician's office with a two-day history of multiple symptoms, including rash on his trunk, headache, fatigue, malaise, myalgias, and high-grade fever. The rash is not associated with pain, itching, or burning. It has expanded over the last two days. He went on a camping trip in Vermont two weeks ago, and recalls a tick bite at that time. There is a single lesion on his trunk, which is erythematous with central clearing. The rest of the examination is unremarkable. What is the most appropriate next step in the management of this patient?
. Perform ELISA for confirmation of Lyme disease
. Perform western blot for confirmation of Lyme disease
. Give him oral doxycycline
. Give him oral amoxicillin
. Give him intravenous ceftriaxone
39) A 35-year-old HIV-positive male comes to the physician's office because of pain on swallowing and substernal burning for the last 3 days. Examination of the oral cavity is unremarkable. Lungs are clear to auscultation and percussion. His current medications include zidovudine, didanosine, indinavir, trimethoprim-sulfamethoxazole, and azithromycin. His last CD4 count is 40cells/microl. What is the most appropriate next step in the management of this patient?
. Esophagoscopy with biopsy, cytology and culture
. Oral Fluconazole
. Oral Famotidine
. Oral Acyclovir
. Oral Ganciclovir
40) A 25-year-old HIV-positive male comes to a physician with complaints of headache and left-sided weakness of recent onset. His temperature is 38°C (100.8°F), blood pressure is 115/70mm Hg, respirations are 14/min, and pulse is 73/min. Neurological examination reveals decreased power, hyperreflexia, and upgoing plantars in the left upper and lower limb. Neuroimaging by CT shows multiple ring-enhancing lesions. What is the most appropriate next step in management?
. Trimethoprim-sulfamethoxazole
. Sulfadiazine and pyrimethamine
. Brain irradiation
. Brain biopsy
. Anti-tuberculous therapy
41) An 18-year-old young man comes to your clinic with complaints of fatigue, malaise, fever, and sore throat for the past few days. His temperature is 38.8°C (101°F), pulse is 90/min, respirations are 18/min, and blood pressure is 135/70 mm Hg. Physical examination reveals cervical lymphadenopathy, pharyngeal hyperemia, and splenomegaly. Complete blood count shows leukocytosis with predominant atypical lymphocytes. The heterophile antibody test is positive. When you ask if he has any questions, he replies with, "Well, I'm a rugby player in school, so I need to know when I can start playing again." What is the most appropriate advice for this patient?
. He can start playing when he becomes afebrile
. He can start playing when he is no longer fatigued
. He can start playing when his labs normalize
. He can start playing when his physical examination is normal
. He can start playing whenever he wants
42) A 25-year-old man presents to the physician's office because of a clenched fist injury ("fight bite") incurred during a gang fight. The injury occurred two days ago and he has now started to develop pain, swelling, and redness around the wound. His immunizations are up to date. His wounds are cleaned in the clinic. Plain films of hand do not show evidence of foreign body or osteomyelitis. Which of the following is the most appropriate antibiotic for his current condition?
. Amoxicillin-clavulanate
. Clindamycin
. Ampicillin
. Erythromycin
. Ciprofloxacin
43) A 30-year-old white, HIV-infected male from Ohio presents with low-grade fever, anorexia and weight loss of 3.6kg (Bib) for the past 3 weeks. His baseline PPD test was negative. His pulse is 87/min, blood pressure is 126/76 mm Hg, temperature is 38.3°C (101°F), and respirations are 18/min. Examination shows oropharyngeal ulcers and hepatosplenomegaly. Lab testing shows: Hematocrit 36%, Platelet count 50,000/microL, WBC count 3,100/microL. Blood culture establishes the diagnosis of histoplasmosis. What is the most appropriate next step in the management of this patient?
. IV amphotericin B followed by lifelong treatment with itraconazole
. IV amphotericin B till the cultures become negative
. Itraconazole till the cultures become negative
. IV amphotericin B plus itraconazole till the cultures become negative
. Lifelong treatment with itraconazole
44) A 7-year-old boy is brought to the office by his mother due to anal pruritus for the past month. His symptoms are most severe at night. Physical examination is normal, except for mild perianal erythema. Stool examination is normal. "Scotch tape test" is positive. Which of the following is the most appropriate treatment for this child's condition?
. Pyrantel pamoate
. Albendazole
. Metronidazole
. Thiabendazole
. Ivermectin
45) A 37-year-old man comes to his primary care physician for the evaluation of slightly pruritic skin lesions around his anus. He denies fever, malaise, and anorexia. He is sexually active with multiple male partners and occasionally uses condoms. He has never been tested for HIV or other sexually transmitted diseases. He has no drug allergies. Examination shows skin-colored, verrucous, papilliform lesions around his anus. Which of the following is the most appropriate treatment for this patient?
. Podophyllin
. Penicillin
. Erythromycin
. Doxycycline
. Tetracycline
46) A 28-year-old, HIV-infected female from Michigan is admitted with Pneumocystis jirovecii pneumonia (PCP) secondary to noncompliance with prophylaxis. She was diagnosed with HIV infection three years ago. Her C04 count on admission is 30/microl, and viral load is 300,000copies/ml. Her pneumonia is adequately treated with IV antibiotics, and she subsequently receives zidovudine, lamivudine, nelfinavir, and trimethoprim-sulfamethoxazole. What is the most appropriate drug to be added to her current regimen?
. Azithromycin
. Rifabutin
. Fluconazole
. Itraconazole
. Ganciclovir
47) A 65-year-old Connecticut resident calls your office after finding a 3 mm red-brown tick attached to his right leg. He just returned from a hiking trip and was about to a take a shower when he discovered the tick. Which of the following is the best advice to give this patient?
. Crush the tick with your fingers
. Remove the tick with tweezers
. Apply petroleum jelly over the tick
. Come to the office tomorrow for tick removal
. Let the tick detach spontaneously
48) A 27-year-old man presents with fever, malaise, anorexia, and fatigue for the last three days. He denies cough, chest pain, arthralgias, and diarrhea. He has history of rheumatic heart disease and recently underwent a dental cleaning. His temperature is 38.5°C (101.3°F), pulse is 90/min, respirations are 18/min, and blood pressure is 135/76 mm Hg. Examination of his fingernail is shown below. Cardiovascular examination reveals an early diastolic murmur in the mitral area. The chest x-ray is negative. Urinalysis shows microscopic hematuria. What is the most appropriate next step in the management of this patient?
. Start antibiotics immediately and then obtain blood cultures
. Start antibiotics after drawing blood for culture
. Do transesophageal echocardiography
. Do transthoracic echocardiography
. Give aspirin and start his penicillin prophvlaxis
49) A 65-year-old African-American man is hospitalized after a car accident. He has a vertebral fracture at the level of the fifth thoracic vertebra and is unable to move his lower extremities. Since his injury, he has needed a catheter for micturition. Which of the following is the most effective measure for decreasing the risk of a urinary tract infection in patients requiring bladder catheterization?
. Antibacterial creams applied to the urethral meatus
. Prophylactic antibiotics
. Antibacterial washes of external genitalia
. Bladder irrigation with antibacterial solutions
. Intermittent catheterization
50) A 32-year-old man from Arkansas presents to physician with a two-day history of fever, headache, malaise, and myalgias. His family says that he seems slightly confused. He recalls having a tick bite two weeks ago after walking through the woods. His temperature is 39°C (102°F), pulse is 90/min, and blood pressure is 125/80 mm Hg. Neck is supple and there is no lymphadenopathy noted. Oropharynx is clear. Chest auscultation is unremarkable. Abdomen is soft and non-tender. There is no rash evident. Neurologic examination is nonfocal. Laboratory testing shows: Complete blood count:Hemoglobin 14.0 g/L, MCV 88 fL, Platelets 78,000/mm3, Leukocyte count 2,500/mm3, Neutrophils 56%, Eosinophils 1%, Lymphocytes 33%, Monocytes 10%. Liver studies:Total protein, serum 6.5 g/dL, Total bilirubin 1.0 mg/dL, Direct bilirubin 0.8 mg/dL, Alkaline phosphatase 110 U/L, Aspartate aminotransferase (SGOT) 98 U/L, Alanine aminotransferase (SGPT) 105 U/L. What is the most appropriate next step in the management of this patient?
. Doxycycline
. Chloramphenicol
. Erythromycin
. Ceftriaxone
. Hepatitis serology
51) A 22-year-old woman presents to the emergency department after she is bitten on her right arm by her neighbor's dog. She provoked the dog while it was eating. The dog is not immunized against rabies, but does not show any signs of rabies. Her right forearm shows a deep bite wound. Her last tetanus booster was 3 years ago. Her wound is cleaned with soap, water, and povidone-iodine solution. What is the most appropriate next step in the management of this patient?
. Kill the dog and do brain biopsy
. Observe the dog for 10 days
. Active immunization for rabies
. Passive immunization for rabies
. Active and passive immunization for rabies
52) A 25-year-old male comes to the clinic because of a painless ulcer on his penis. He denies fever or urethral discharge, but admits to recent sexual activity with a prostitute. He describes severe rash and face swelling with penicillin. Physical examination reveals a shallow, non-tender ulcer. There is no lymphadenopathy. Darkfield microscopy reveals spirochetes. Which of the following is the most appropriate treatment for this patient?
. Intramuscular benzathine penicillin
. Ciprofloxacin
. IV aqueous crystalline penicillin
. Oral clindamycin
. Oral doxycycline
53) A 40-year -old man who underwent a renal transplant six months ago comes to the clinic with fever, chills, and a productive cough. His temperature is 39.4°C (103°F), pulse is 110/min, respirations are 22/min, and blood pressure is 110/65 mmHg. Chest x-ray shows a right lower lobe nodule with a cavity. Sputum gram stain is shown below. What is the most appropriate treatment of this patient's condition?
. Penicillin
. Trimethoprim-sulfamethoxazole
. Vancomycin
. Gentamycin
. Metronidazole
54) A 36-year-old woman who lives in the suburbs of a large city comes to your office for a tuberculin skin test. She will be volunteering in her daughter's school cafeteria and the school district requires tuberculin testing. You inject a small amount of Mycobacterium tuberculosis purified protein derivative (PPD) in the skin and 2 days later she returns for a reading. You measure 12 mm of induration. She reports no history of tuberculosis exposure and no underlying medical conditions. She has never before been tested for tuberculosis. She was born in the United States, is not a healthcare worker, and has never spent time in prison. What is the best next step in her management?
. Chest X-ray
. Observation
. Isoniazid for 6 months
. Isoniazid with pyridoxine for 9 months
. Isoniazid, rifampin and pyrazinamide for 8 weeks
55) A 22-year-old female presents to the office with a three-day history of rash, fever, and malaise. There is no burning or itching associated with the rash. Two weeks ago, she had been camping in northern Massachusetts, and noted a tick bite after walking through the woods. She is twelve weeks pregnant. The rash is shown below. The examination is otherwise unremarkable. What is the most appropriate treatment for this patient?
. Doxycycline
. Amoxicillin
. Azithromycin
. Ceftriaxone
. Penicillin G
56) A 5-year-old Hispanic girl is brought to the emergency department (ED) due to a cat bite on her right upper extremity. She was bitten after rambunctiously playing for several minutes with the cat, which had just been given to her as a birthday gift. Her most recent booster tetanus vaccine was one year ago. Her temperature is 36.7°C (98°F), blood pressure is 108/70 mm Hg, pulse is 107/min, and respirations are 21/min. Physical examination reveals a deep puncture wound on the volar aspect of her right forearm. Before going to the ED, her mother cleaned the wound with hydrogen peroxide. There is no visible debris in the wound, and little bleeding is evident. Neurovascular function is intact. Her wound is cleaned in the ED with Betadine and lavaged with saline solution. What is the best next step in managing this girl's care?
. Bandage with dry gauze and discharge home
. Prescribe amoxicillin
. Prescribe amoxicillin/clavulanate
. Prescribe erythromycin
. Prescribe levofloxacin
57) A 57-year-old male comes to your office in the middle of January. He complains of a 4-day history of a nonproductive cough and coryza. He also has vague muscle aches and a mild headache. He denies any shortness of breath or chest pain. His past medical history is significant for hyperlipidemia and impaired glucose tolerance. He takes aspirin and simvastatin 40 mg at bedtime. He has no known drug allergies. His medical records show that he did not show up for his scheduled annual influenza vaccine this year. His temperature is 38.5°C (101.3°F), blood pressure is 135/80 mm Hg, and pulse is 88/min. Physical examination shows conjunctival redness and an erythematous oropharynx. The tympanic membranes are clear. The heart sounds are audible with no added murmurs, rubs or gallops. His breath sounds are vesicular in quality and equal bilaterally. Which of the following is the most appropriate next step in management?
. Administer influenza vaccination now
. Write a prescription for amantidine
. Write a prescription for oseltamivir
. Advise bed rest and symptomatic treatment with acetaminophen
. Empiric trial of oral antibiotics
58) A 60-year-old Caucasian woman undergoes elective coronary artery bypass surgery and aortic valve replacement. Her postoperative course is complicated by acute renal failure, atrial fibrillation, and pulmonary edema. On the third postoperative day, extubation is attempted but not tolerated, thus warranting reintubation. On the fifth postoperative day, she develops a fever to 38.9°C (102°F). Her pulse is 110-120/min and irregular, respirations are 36/min, and blood pressure is 110/65 mmHg. Her chest x-ray shows right middle and lower lobe infiltrates. WBC count is elevated with bandemia. Gram stain of her sputum shows gram-negative rods. She is given intravenous ceftriaxone; however, she deteriorates over the next 24 hours. Which of the following is the most appropriate next step in the management of this patient?
. Stop ceftriaxone and start piperacillin-tazobactam
. Continue ceftriaxone and add vancomycin
. Stop ceftriaxone and start clindamycin
. Continue ceftriaxone and add ceftazidime
. Continue ceftriaxone and add amphotericin
59) A 34-year-old man is traveling in Southeast Asia on business. He is staying in Western-style hotels and eating food in large restaurants. He has not eaten from street vendors. One week after arrival, he develops symptoms of anorexia, nausea, and abdominal cramps followed by the sudden onset of watery diarrhea. He has no fever or chills and there is no blood or pus in the stools. Which of the following is the most appropriate therapy for his condition?
Amoxicillin
Symptomatic therapy with loperamide
Doxycycline
Oral rehydration only
Specific antitoxin
60) A 45-year-old woman is undergoing chemotherapy for breast cancer. She presents 10 days after her last chemotherapy with fever (temperature > 38.5°C), but no other symptoms except a sore throat and mouth. On examination, she looks well, there is oral mucositis, ears are normal, lungs are clear, and the central line site is clean. The CXR, urinalysis, and biochemistry are normal. Her WBC is 800/mL and the absolute neutrophil count is low ( < 500). Which of the following is the most appropriate next step in management?
Start empiric bacterial antibiotics
Start empiric antifungal and bacterial antibiotics
Acetaminophen alone until culture results are available
Start antiviral medications for HSV-1
Start antiviral and bacterial antibiotics
61) A 74-year-old man residing in a nursing home develops symptoms of high fever, diarrhea, chest pain, and nonproductive cough. His temperature is 40°C, blood pressure 120/80 mm Hg, respiration 24/min, and oxygen saturation 90%. He has bibasilar crackles, normal heart sounds, and a soft nontender abdomen. His CXR reveals bilateral lower lobe infiltrates. He is not able to provide any sputum, and the urine is positive for legionella antigen. Which of the following is the most appropriate antibiotic choice?
Trimethoprim-sulfamethoxazole
Azithromycin
Ceftriaxone
Cefuroxime
Gentamicin
62) A 28-year-old female presents to her internist with a 2-day history of low-grade fever and lower abdominal pain. She denies nausea, vomiting, or diarrhea. On physical examination, there is temperature of 38.3°C (100.9°F) and bilateral lower quadrant tenderness, without point or rebound tenderness. Bowel sounds are normal. On pelvic examination, an exudate is present and there is tenderness on motion of the cervix. Her white blood cell count is 15,000/μL and urinalysis shows no red or white blood cells. Serum β-hCG is undetectable. Which of the following is the best next step in managemen
. Treatment with ceftriaxone and doxycycline
. Endometrial biopsy
. Surgical exploration
. Dilation and curettage
. Aztreonam
63) Two students from a university dormitory building have contracted meningitis due to Neisseria meningitides. Which of the following students in the dormitory are most likely to benefit from chemoprophylaxis?
Everybody in the dormitory, with oral amoxicillin
Close contacts only, with oral amoxicillin
Everybody in the dormitory, with oral rifampin
Close contacts only, with oral rifampin
Everybody in the dormitory, with meningococcal vaccine
64) You are a physician in charge of patients who reside in a nursing home. Several of the patients have developed influenza-like symptoms, and the community is in the midst of influenza A outbreak. None of the nursing home residents have received the influenza vaccine. Which course of action is most appropriate?
. Give the influenza vaccine to all residents who do not have a contraindication to the vaccine (ie, allergy to eggs)
. Give the influenza vaccine to all residents who do not have a contraindication to the vaccine; also give oseltamivir for 2 weeks to all residents
. Give amantadine alone to all residents
. Give azithromycin to all residents to prevent influenza-associated pneumonia
. Do not give any prophylactic regimen
65) A 22-year-old university student complains of fatigue and malaise for the past 2 weeks. She also reports feeling feverish, and recently had a sore throat. Physical examination reveals enlarged tonsils and palpable cervical lymph nodes. There is also tenderness in the right upper quadrant on deep palpation, and minimal splenomegaly. Laboratory data show hemoglobin 13 g/dL; hematocrit 40%; platelets 340,000/mL; WBC 9400/mL, with 35% segmented neutrophils, 1% eosinophils, and 64% lymphocytes, of which 36% were atypical. A heterophil antibody (sheep cell agglutination) test is positive. Which of the following is the most appropriate initial treatment for this condition?
Gamma-globulin
Adequate rest
Chlorambucil
Chloramphenicol
Radiation therapy
66) A 44-year-old renal transplant patient develops severe cough and shortness of breath on exertion. On examination, he appears dyspneic, respirations 24/min, pulse 110/min, and oxygen saturation 88%. His lungs are clear on auscultation and heart sounds are normal. CXR shows bilateral diffuse perihilar infiltrates. Bronchoscopy and bronchial brushings show clusters of cysts that stain with methenamine silver. Which of the following is the most appropriate next step in management?
Amphotericin B
Cephalosporins
Trimethoprim-sulfamethoxazole
Aminoglycosides
Penicillins
67) A businesswoman needs to make frequent trips to South America, but every time she is there, she develops traveler’s diarrhea, which requires her to change her business schedule. To prevent future episodes during business trips, she is inquiring about prophylaxis methods. Which of the following is the most helpful advice for her?
Take loperamide for symptoms
Take trimethoprim-sulfamethoxazole every day
Take azithromycin every day
Take doxycycline every day
Take ciprofloxacin only if moderate or severe symptoms develop
68) A 30-year-old male patient complains of fever and sore throat for several days. The patient presents to you today with additional complaints of hoarseness, difficulty breathing, and drooling. On examination, the patient is febrile and has inspiratory stridor. Which of the following is the best course of action?
. Begin outpatient treatment with ampicillin
. Culture throat for β-hemolytic streptococci
. Admit to intensive care unit and obtain otolaryngology consultation
. Schedule for chest x-ray
. Obtain Epstein-Barr serology
69) A 35-year-old previously healthy male develops cough with purulent sputum over several days. On presentation to the emergency room, he is lethargic. Temperature is 39°C, pulse 110, and blood pressure 100/70. He has rales and dullness to percussion at the left base. There is no rash. Flexion of the patient’s neck when supine results in spontaneous flexion of hip and knee. Neurologic examination is otherwise normal. There is no papilledema. A lumbar puncture is performed in the emergency room. The cerebrospinal fluid (CSF) shows 8000 leukocytes/μL, 90% of which are polys. Glucose is 30 mg/dL with a peripheral glucose of 80 mg/dL. CSF protein is elevated to 200 mg/dL. A CSF Gram stain shows gram-positive diplococci. Which of the following is the correct treatment option?
. Begin acyclovir for herpes simplex encephalitis
. Obtain emergency MRI scan before beginning treatment
. Begin ceftriaxone and vancomycin for pneumococcal meningitis
. Begin ceftriaxone, vancomycin, and ampicillin to cover both pneumococci and Listeria
. Begin high-dose penicillin for meningococcal meningitis
70) A young man has recently been bitten by a stray dog. He has a penetrating wound to the right forearm. The dog is nowhere to be found. In the emergency room, the wound is cleaned with water and povidone-iodine solution. Which of the following is the most appropriate next step in management?
Start postexposure prophylaxis
Contact the local public health professional for further advice
Treat with oral doxycycline
Treat with IV ceftriaxone
Start IV acyclovir
71) A 22-year-old male, recently incarcerated and now homeless, has received one week of clarithromycin for low-grade fever and left upper-lobe pneumonia. He has not improved on antibiotics, with persistent cough productive of purulent sputum and flecks of blood. Repeat chest x-ray suggests a small cavity in the left upper lobe. Which of the following statements is correct?
. The patient has anaerobic infection and needs outpatient clindamycin therapy
. The patient requires sputum smear and culture for acid fast bacilli
. The patient requires glove and gown contact precautions
. Isoniazid prophylaxis should be started if PPD is positive
. Drug resistant pneumococci may be causing this infection
72) A 23-year-old woman visits your office because of headache, malaise, anorexia, pain in both sides of her jaw, and discomfort in both lower abdominal quadrants. Physical examination reveals enlarged parotid glands; bilateral lower quadrant abdominal tenderness; a temperature of 38.7°C; and a pulse rate of 92/min. Serologic testing (IgM) confirms the diagnosis of mumps. Which of the following is the most appropriate treatment for this condition?
Symptomatic
Immunization
Broad-spectrum antibiotics
Sulfonamides
Steroids
73) A 25-year-old woman complains of dysuria, frequency, and suprapubic pain. She has not had previous symptoms of dysuria and is not on antibiotics. She is sexually active and on birth control pills. She has no fever, vaginal discharge or history of herpes infection. She denies back pain, nausea, or vomiting. On physical examination she appears well and has no costovertebral angle tenderness. A urinalysis shows 20 white blood cells per high power field. Which of the following statements is correct?
. A 3-day regimen of trimethoprim-sulfamethoxazole is adequate therapy
. Quantitative urine culture with antimicrobial sensitivity testing is mandatory
. Obstruction resulting from renal stone should be ruled out by ultrasound
. Low-dose antibiotic therapy should be prescribed while the patient remains sexually active
. The etiologic agent is more likely to be sensitive to trimethoprim-sulfamethoxazole than to fluoroquinolones
74) A 40-year-old woman cut her finger while cooking in her kitchen. Two days later she became rapidly ill with fever and shaking chills. Her hand became painful and mildly erythematous. Later that evening her condition deteriorated as the erythema progressed and the hand became a dusky red. Bullae and decreased sensation to touch developed over the involved hand. What is the most important next step in the management of this patient?
. Surgical consultation and exploration of the wound
. Treatment with clindamycin for mixed aerobic-anaerobic infection
. Treatment with penicillin for clostridia infection
. Vancomycin to cover community-acquired methicillin-resistant Staphylococcus
. Evaluation for acute osteomyelitis
75) A 60-year-old man comes to the physician because of a 2-week history of worsening fatigue. He has chronic renal insufficiency, hypertension, diabetes mellitus, hypercholesterolemia, hypothyroidism, polymyalgia rheumatica and depression. He was started on lisinopril for the prevention of proteinuria from diabetic nephropathy. Physical examination shows a few basal crackles. He is being considered for dialysis. Laboratory studies show hyperkalemia with serum K + of 6.0 mEq/L. EKG shows no abnormalities. Which of the following is the best treatment to remove K+ from his body?
. Sodium bicarbonate
. Beta agonists
. Calcium gluconate
. Kayexalate
. Insulin plus glucose
76) A 45-year-old man with advanced chronic renal failure comes to the physician because of edema of his feet. His temperature is 37°C (99°F), blood pressure is 150/100mm Hg, pulse is 78/min, and respirations are 15/min. Examination shows bilateral ankle edema. Laboratory studies show BUN of 62 mg/dl, serum creatinine of4.2 mg/dl, serum potassium of 5.6 meq/l, serum sodium of 146 meq/l, and total plasma cholesterol of 260 mg/dl. Which of the following is most likely to improve the prognosis of his disease?
. Captopril
. Simvastatin
. Protein restriction
. Salt restriction
. Potassium restriction
77) A 68-year-old male presents to the emergency room with cough. Chest x-ray is clear of infiltrates but reveals a right upper lobe lung lesion incidentally. A chest CT scan with IV contrast is performed in the emergency department and reveals a 1 cm x 2cm round lesion in the right upper lobe. The patient is admitted to the hospital, and by day 3 of his hospitalization, he has developed acute renal failure. The patient's past medical history is significant for hypertension, hyperlipidemia, diabetes and coronary artery disease. His medications include aspirin, hydrochlorothiazide, clopidogrel, metoprolol, atorvastatin and lisinopril, all of which he has been taking for several years. He has no known drug allergies. He is a former smoker and does not use alcohol or drugs. His baseline blood pressure is 140/90 mm Hg and has remained in the 140s/90s throughout this admission. Physical examination is otherwise unremarkable. The patient's laboratory values from the time of admission to present are given below. Over the course of the next week, the patient's renal function returns to normal. Which of the following would most likely have prevented his renal failure?
. Prednisone
. Furosemide
. Stopping clopidogrel
. Intravenous hydration
. 100% oxygen mask
78) A 23-year-old male hospitalized for confusion and seizures is treated with intravenous high-dose acyclovir. On the third day of hospitalization, his serum creatinine level increases to 3.4 mg/dl from a baseline of 0.9 mg/dl at admission. The observed finding could have been potentially prevented by which of the following?
. Careful allergy history taking
. Monitoring the blood drug levels
. Pre-treatment with allopurinol
. Pre-treatment with prednisone
. Aggressive intravenous hydration
79) A 27-year-old man comes to the physician because of a 1-day history of fever and joint pains. He is being treated with cephalexin for a skin infection. His urine has turned darker. His temperature is 38.5°C (101.3°F), blood pressure is 125/70 mm Hg, pulse is 90/min, and respirations are 15/min. Examination shows a skin rash; examination otherwise shows no abnormalities. Urinalysis shows: 8 RBCs/HPF, 12 WBCs/HPF with white cell casts, eosinophiluria, and a mild degree of proteinuria. Laboratory studies show a BUN of 40 mg/dl and serum creatinine of 2.2 mg/dl. Which of the following is the most appropriate next step in management?
. Discontinue cephalexin
. Start ampicillin and gentamicin
. Start oral ciprofloxacin
. Start intravenous steroids
. Start oral steroids
80) A 30-year-old man is admitted to the hospital with a diagnosis of acute renal failure secondary to poststreptococcal glomerulonephritis. On his fifth hospital day, he develops retrosternal, non-radiating chest pain which is relieved by leaning forward. He denies the use of tobacco or drugs. He drinks alcohol occasionally. He has no past history of any serious illness. His temperature is 37.6°C (99.8°F), blood pressure is 145/95 mm Hg, pulse is 80/min, and respirations are 20/min. A pericardial friction rub is heard on chest auscultation. The rest of the examination shows no abnormalities. EKG shows ST segment elevation in all leads, with elevation of the PR segment in lead aVR. The chest x-ray is normal. Urinalysis shows hematuria, red cell casts and mild proteinuria. Laboratory studies show a BUN level of 60 mg/dl and a serum creatinine level of 3 8 mg/dl. What is the most appropriate next step in management?
. Broad spectrum antibiotics
. Intravenous steroids
. Hemodialysis
. Thrombolytic therapy
. NSAIDs
81) A 47-year-old diabetic woman comes to the physician due to the recent onset of tremors. She has undergone combined pancreatic and kidney transplantation secondary to end stage renal disease and diabetes. She takes multiple medications, including immunosuppressants. Her temperature is 36.1° C (97°F), blood pressure is 152/90 mm Hg, pulse is 78/min, and respirations are 16/min. Examination shows gum hypertrophy. Laboratory studies show: Hb 13.0 g/dl, WBC 8,000/cmm, Serum Na 135 mEq/L, Serum K 5.3 mEq/L, BUN 26 mg/dl, Serum Creatinine 1.7 mg/dl. Which of the following immunosuppressants is most likely responsible for her presentation?
. Tacrolimus
. Cyclosporine
. Azathioprine
. Mycophenolate
. Diuretic
82) A 45-year-old male patient comes to the physician's office for a routine check-up. He denies any symptoms and says he feels "perfectly healthy." He was diagnosed with hypertension and mixed hypercholesterolemia a year ago. He is currently taking hydrochlorothiazide, amiloride and simvastatin daily. He does not use tobacco, alcohol or drugs. His blood pressure today is 135/85 mm Hg. Physical examination shows no abnormalities. Routine blood results reveal the following: CBC: Hb 14.2g/dL, Ht 42%, MCV 86 fl, Platelet count 260,000/cmm, Leukocyte count 8,500/cmm, Neutrophils 70%, Eosinophils 1%, Lymphocytes 24%, Monocytes 5%. Serum: Serum Na 140 mEq/L, Serum K 5.7 mEq/L, Chloride 100 mEq/L, Bicarbonate 24 mEq/L, BUN 10 mg/dL, Serum Creatinine 1.1 mg/dL, Calcium 9.0 mg/dL, Blood Glucose 118 mg/dL, Total cholesterol 220 mg/dL, LDL cholesterol 130 mg/dL. The blood sample is checked and is not hemolysed. The EKG shows normal sinus rhythm. What is the most appropriate next step in the management of this patient?
. Intravenous calcium gluconate
. Intravenous dextrose + insulin
. Stop amiloride and recheck lab results in 1 week
. Stop HCTZ and recheck lab results in 1 week
. Start patient on a low potassium diet
83) A 50-year-old man comes to the emergency department due to a sudden onset of severe, colicky pain in the right flank. He was admitted twice in the past for similar complaints; he was managed conservatively and sent home on both occasions. He has no other medical problems. He does not use tobacco, alcohol or drugs. His vital signs are stable. He is given IV fluids and narcotics. Laboratory studies show: Hb 14.5 g/dL, WBC 13,000/cmm; no bands, Platelets 300,000/cmm, BUN 16 mg/dl, Serum Creatinine 0.8 mg/dl. CT scan of the abdomen without contrast shows renal calculi. Which of the following is the best advice for the prevention of future stones in this patient?
. Decrease dietary calcium intake
. Mega doses of Vitamin C
. Decrease dietary protein and oxalate
. Restrict fluid intake
. Increase sodium intake
84) A 64-year-old male with a past medical history of hypertension, diabetes and chronic renal insufficiency presents with gross hematuria. His baseline serum creatinine is 1.6-1.7 mg/dl. The patient's medications include aspirin, hydrochlorothiazide, enalapril, and simvastatin. He has no known environmental, medication, or contrast allergies. On physical examination, the patient has a blood pressure of 130/80 mm Hg. The examination is otherwise unremarkable. Contrast CT scan of the abdomen is planned to evaluate his condition. Which of the following interventions would be most helpful in preventing contrast-induced kidney damage?
. Prednisone
. Non-ionic contrast agent
. Furosemide
. 100% oxygen mask
. Stopping simvastatin
85) A 26-year-old man comes to the emergency department because of a sudden onset of severe right-sided flank pain. The pain is colicky and radiates from the flank to the scrotum. He also has nausea, vomiting and dark-colored urine. He has never had these symptoms before. His temperature is 37°C (98.6°F), blood pressure is 126/70 mm Hg, pulse is 90/min, and respirations are 18/min. Examination shows no abnormalities. He is given adequate analgesia. Non-contrast helical CT shows a 4 mm radiopaque stone in the right upper ureter. Laboratory studies show serum calcium of 9.8 mg/dl, serum creatinine of 0.9 mg/dl, and BUN of 15mg/dl. Urinalysis shows hematuria but no casts. Which of the following is the most appropriate next step in management?
. 24 hr urine collection for metabolic evaluation
. Reassurance
. Fluid intake greater than 2L/day
. Intake of potassium citrate
. Restriction of dietary oxalate
86) A healthy 54-year-old man comes to the physician for a routine health maintenance examination. He has no complaints, but he is requesting for a CT scan of the abdomen. His father died at the age of 60 due to the sudden rupture of an undiagnosed abdominal aortic aneurysm. He has a history of hypertension and gouty arthritis. His social history is not significant. His vital signs are stable. Physical examination shows no abnormalities. ACT scan of the abdomen is shown below. Which of the following is the most appropriate next step in management?
. Reassurance
. Surgery
. CT guided biopsy
. Antibiotics
. Repeat CT scan in 3 months
87) A 72-year-old woman with poorly controlled type 2 diabetes mellitus presents to your clinic one week after being discharged from the hospital. She had been admitted with pyelonephritis secondary to a multi-drug resistant organism, and received several days of intravenous antibiotics. Her serum creatinine on admission had been 2.1 mg/dl. Today it is found to be 4.9 mg/dl. Urinalysis reveals rare epithelial casts and no white blood cells. FENa is greater than 2%. What antibiotic did she most likely receive during her hospitalization?
. Nafcillin
. Vancomycin
. Levofloxacin
. Amikacin
. Doxycycline
88) A 58-year-old man comes to the physician and complains of "problems with erection." He has recurrent and persistently painful erections. His other medical problems include ulcerative colitis, kidney stones, insomnia, depression, hypertension, drug-induced diabetes, obesity and hypercholesterolemia. He does not use tobacco, alcohol, or drugs. He takes prednisone, mesalamine, insulin, 6-mercaptopurine, simvastatin, glyburide, enalapril, trazodone, and fluoxetine. He has no known drug allergies. His vital signs are stable. The general physical examination is unremarkable. Avoidance of which of the following medications could have prevented his condition?
. Fluoxetine
. Trazodone
. Enalapril
. Glyburide
. Simvastatin
89) A 30-year-old woman comes to the physician due to the recent onset of occipital headaches. She has taken acetaminophen several times, but the pain returns. She has no fever or visual problems. She has not had similar episodes in the past. She has no history of serious illness. Her temperature is 36.1°C (98°F), blood pressure is 160/90 mm Hg, pulse is 88/min, and respirations are 16/min. Physical examination shows a rightsided renal bruit Which of the following is the most appropriate treatment for this patient's condition?
. ACE inhibitors
. Furosemide
. Angioplasty with stent placement
. Surgery
. Oral prednisone
90) A 45-year-old man with known cirrhosis due to hepatitis C is admitted to the hospital for abdominal discomfort and confusion. Physical examination reveals a distended abdomen, leg edema, and deep yellow discoloration of the skin and sclerae. Bibasilar crackles are heard on chest auscultation. His serum sodium level is 127 mEq/L, potassium level is 2.9 mEq/L, and creatinine level is 1.3 mg/dl. On day 3 of his hospitalization, the serum creatinine is 4.2mg/dl. A urinalysis reveals: Protein negative, Glucose negative, WBC 4-5/hpf, RBC 0-1/hpf. Renal ultrasound is normal and the post-void residual urinary volume is less than 50 ml. He is given 2 L of normal saline intravenously with no change in his serum creatinine concentration. This patient's kidney dysfunction can be best corrected by which of the following?
. High-dose spironolactone
. ACE inhibitors
. Liver transplantation
. Broad spectrum antibiotics
. Pegylated interferon
91) A 56-year-old man develops oliguria three days after having a kidney transplantation. His postoperative course was uncomplicated. His blood pressure is 160/100 mm Hg and heart rate is 90/min. Palpation of the transplant reveals mild tenderness. Laboratory studies show: Serum sodium 145 mEq/L, Serum potassium 5.5 mEq/L, Serum calcium 8.6 mg/dl, Serum creatinine 3.2 mg/dl, BUN 30 mg/dl. His serum cyclosporine level is normal. Renal ultrasonography does not detect dilatation of the calyces. Biopsy of the transplant shows heavy lymphocyte infiltration and vascular involvement with swelling of the intima. Which of the following is the most appropriate next step in management?
. Decrease the dose of cyclosporine
. Give IV steroids
. Order ureterography
. Administer IV diuretics
. Prepare for surgery
92) A 25-year-old woman comes to the physician because of a 3-day history of burning micturition and increased urinary frequency. She has suprapubic discomfort. She denies having unusual vaginal discharge. She has been sexually active and monogamous for the past 4 years with her husband. Her temperature is 37.1°C (98.9°F), blood pressure is 110/70 mm Hg, pulse is 68/min, and respirations are 15/min. Examination shows suprapubic tenderness without flank tenderness. The rest of the examination is normal. Urinalysis shows: Specific gravity 1.020, Blood Trace, Glucose Negative, Ketones Negative, Leukocyte esterase Positive, Nitrites Positive, WBC 40-50/hpf, RBC 6-10/hpf, Bacteria 50+. Which of the following is the most appropriate next step in management?
. Urine culture
. Oral trimethoprim-sulfamethoxazole
. Oral ciprofloxacin
. Oral nitrofurantoin
. Intravenous trimethoprim-sulfamethoxazole
93) A 34-year-old man is being evaluated for possible end-stage renal disease. He has a long history of diabetes, type 1. He previously developed chronic renal insufficiency despite being on enalapril and insulin. His renal function is getting worse day by day. A nephrologist is currently managing his renal condition. Which of the following long-term treatments would give the best survival rate for this patient?
. Hemodialysis
. Peritoneal dialysis
. Renal transplantation from a cadaver
. Renal transplantation from a living related donor
. Renal transplantation from a living unrelated donor
94) A 50-year-old diabetic woman presents for follow-up of her hypertension. Her blood pressure is 152/96 in the office today and she brings in readings from home that are consistently in the same range over the past month. Her current medications are amlodipine 5 mg daily and hydrochlorothiazide 25 mg daily. The diuretic was added when she developed peripheral edema on the amlodipine; now she has only trace peripheral edema. A spot urine specimen shows 280 μg of albumin per mg creatinine (microalbuminuria is present if this value is between 30 and 300 μg/mg). What would be the best next therapeutic step in this patient?
. Add clonidine
. Add a beta-blocker
. Increase the thiazide diuretic dose
. Add an alpha-blocker
. Add angiotensin-converting enzyme inhibitor or angiotensin receptor blocker
95) A 64-year-old woman develops severe diarrhea 2 weeks after finishing antibiotics for pneumonia. She has also noticed decreased urine output despite drinking lots of fluids. On examination, she has a postural drop in her blood pressure, the JVP is low, and the abdomen is soft but diffusely tender. Despite giving 4 L of normal saline, her urine output remains low. The urinalysis is positive for heme-granular casts and the urine sodium is 42mEq/L. Which of the following medications should be held during the recovery phase of this woman’s ARF?
Acetaminophen
Digoxin
Lorazepam
Enalapril
Simvastatin
96) A 67-year-old man with a history of gout presents with intense pain in his right great toe. He has a complex past medical history, including hypertension, coronary artery disease, congestive heart failure, myelodysplasia, and chronic kidney disease with a baseline creatinine of 3.2 mg/dL and a uric acid level of 10 mg/dL. His medications include aspirin, simvastatin, clopidogrel, furosemide, amlodipine, and metoprolol. What is the best therapy in this situation?
. Colchicine 1.2 mg po initially, followed by 0.6 mg 1 hour later
. Allopurinol 100 mg po daily and titrate to uric acid less than 6 mg/dL
. Prednisone 40 mg po daily
. Naproxen 750 mg po once followed by 250 mg po tid
. Probenecid 250 mg po bid
97) A 60-year-old diabetic woman develops angina and will need a coronary angiogram for evaluation of coronary artery disease. She has a creatinine of 2.2. Which of the following is the most effective in reducing the risk of contrast induced nephropathy?
. Administer mannitol immediately after the contrast is given
. Perform prophylactic hemodialysis after the procedu
. Give IV hydration with normal saline or sodium bicarbonate prior to and following the procedure
. Indomethacin 25 mg the morning of the procedure
. Dopamine infusion before and after the procedure
98) A 56-year-old man presents with hypertension and peripheral edema. He is otherwise healthy and takes no medications. Family history reveals that his father and a brother have kidney disease. His father was on hemo-dialysis before his death at age 68 of a stroke. Physical examination reveals BP 174/96 mm Hg and AV nicking on funduscopic examination. He has a soft S4 gallop. Bilateral flank masses measuring 16 cm in length are palpable. Urinalysis shows 15 to 20 RBC/hpf and trace protein but is otherwise normal; his serum creatinine is 2.4 mg/dL. Which is the most likely long-term complication of his condition?
. End-stage renal disease requiring dialysis or transplantation
. Malignancy
. Ruptured cerebral aneurysm
. Biliary obstruction owing to cystic disease of the pancreas
. Dementia
99) A 63-year-old woman has Type II diabetes mellitus, which is well-controlled. Her physical examination is positive for peripheral neuropathy in the feet and nonproliferative retinopathy. A urinalysis is positive for proteinuria. Which of the following treatments is most likely to attenuate the course of renal disease?
Calcium channel blockers
ACE inhibitors
Hepatic hydroxymethylglutaryl- coenzyme A (HMG-CoA) inhibitors
Dietary carbohydrate restriction
Weight reduction
100) A 60-year-old man is brought in by ambulance and is unable to speak. The EMS personnel tell you that a neighbor informed them he has had a stroke in the past. There are no family members present. His serum sodium is 118 mEq/L. Which of the following is the most helpful first step in the assessment of this patient’s hyponatremia?
. Order a chest x-ray
. Place a Foley catheter to measure 24-hour urine protein
. Clinical assessment of extracellular fluid volume status
. CT scan of head
. Serum AVP (arginine vasopressin) level
101) A 39-year-old woman is admitted to the gynecology service for hysterectomy for symptomatic uterine fibroids. Postoperatively the patient develops an ileus accompanied by severe nausea and vomiting; ondansetron is piggybacked into an IV of D5 ½ normal saline running at 125 cc/h. On the second postoperative day the patient becomes drowsy and displays a few myoclonic jerks. Stat labs reveal Na 118, K 3.2, Cl 88, HCO3 22, BUN 3, and creatinine 0.9. Urine studies for Na and osmolality are sent to the lab. What is the most appropriate next step?
. Change the IV fluid to 0.9% (normal) saline and restrict free-water intake to 600 cc/d
. Change the ondansetron to promethazine, change the IV fluid to lactated Ringer solution, and recheck the Na in 4 hours
. Start 3% (hypertonic) saline, make the patient NPO, and transfer to the ICU
. Change the IV fluid to normal saline and give furosemide 40 mg IV stat
. Make the patient NPO and send for stat CT scan of the head to look for cerebral edema
102) You evaluate a 48-year-old man for chronic renal insufficiency. He has a history of hypertension, osteoarthritis, and gout. He currently has no complaints. His medical regimen includes lisinopril 40 mg daily, hydro-chlorothiazide 25 mg daily, allopurinol 300 mg daily, and acetaminophen for his joint pains. He does not smoke but drinks 8 oz of wine on a daily basis. Examination shows BP 146/86 mm Hg, pulse 76, a soft S4 gallop, and mild peripheral edema. There is no abdominal bruit. His UA reveals 1+ proteinuria and no cellular elements. Serum creatinine is 2.2 mg/dL and his estimated GFR from the MDRD formula is 42 mL/minute. What is the most important element is preventing progression of his renal disease?
. Discontinuing all alcohol consumption
. Discontinuing acetaminophen
. Adding a calcium channel blocker to improve blood pressure control
. Obtaining a CT renal arteriogram to exclude renal artery stenosis
. Changing the lisinopril to losartan
103) A 45-year-old woman with cirrhosis secondary to autoimmune hepatitis is seeing her hepatologist for routine follow-up. She reports that she has been feeling relatively well lately, and complains only of mild fatigue. Her medical history is otherwise unremarkable. Her medications include lactulose, spironolactone, propranolol and furosemide. Her vital signs are stable and the remainder of her physical exam is consistent with compensated cirrhosis. Her laboratory values are given below: Sodium 132 mEq/L, Potassium 4.1 mEq/L, Chloride 100 mEq/L, Bicarbonate 24 mEq/L, Glucose 102 mg/dl, Creatinine 0.9 mg/dl, Calcium 7.4 mg/dl, Total protein 6.1 g/dl, Albumin 2.5 g/dl, Total bilirubin 2.1 mg/dl, AST 80 units/L, ALT 102 units/L. What is the most appropriate next step in the management of this patient's hypocalcemia?
. Calcium gluconate infusion
. Measurement of vitamin D levels
. Replace furosemide with hydrochlorothiazide (HCTZ)
. Initiate oral calcium and vitamin D replacement
. No intervention is required
104) A 43-year-old female with history of hepatitis C, alcohol use and cirrhosis is admitted to the hospital for severe vomiting for the last 2 days. Her temperature is 36.7°C (98.2°F), pulse is 90/min, respirations are 14/min and blood pressure is 100/70 mmHg supine. Her lungs are clear to auscultation. She has mild epigastric tenderness, but there is no abdominal distention. Extremities have no edema. Her laboratory profile shows: Blood PH 7.49, PaO2 100 mmHg, PaCO2 41 mm Hg, HCO3- 30 mEq/L, Sodium 138 mEq/L, Potassium 3.0 mEq/L, Chloride 95 mEq/L. Which of the following is the most appropriate treatment for the patient's alkalosis?
. Intravenous ammonium chloride and potassium
. Intravenous hydrochloric acid and potassium
. Intravenous mannitol
. Intravenous normal saline and potassium
. Dialysis
105) A 21-year-old woman complains of progressive weakness and loss of energy. She nearly collapsed yesterday while performing one of her routine 3 hour workouts. Additionally, she has been performing badly in college despite persistent attempts to improve her grades. Physical exam reveals a blood pressure of 102/58 mmHg, heart rate of 113/min, fine hair covering her face, and normal heart and lungs. Laboratory studies show the following findings: Sodium 140 mEq/L, Potassium 24 mEq/L, Calcium 10.1 mg/dL, Chloride 90 mEq/L, Bicarbonate 40 mEq/L, Urine chloride 14 mEq/L. Which of the following is most likely to correct the laboratory abnormalities in this patient?
. Calcium gluconate infusion
. Normal saline infusion
. Sodium bicarbonate solution infusion
. Loop diuretics
. Hyperventilation
106) A 52-year-old man with Burkitt lymphoma is admitted to the hospital for chemotherapy. He is started on chemotherapy without incident. Two days into his hospitalization, he develops new peaked T-waves on his ECG. Laboratory analysis reveals a serum potassium concentration of 6.2 mEq/L, and the patient is given one ampule of calcium gluconate emergently. His renal function is within normal limits. Which of the following additional interventions will most rapidly correct his hyperkalemia?
. Hemodialysis
. Sodium polystyrene sulfonate
. Furosemide
. Insulin and glucose
. High-dose inhaled β2 agonist
107) A 58-year-old alcoholic is admitted with diagnosis of decompensated liver cirrhosis. The lab panel is shown below on the day of admission and 3 days later. Which of the following best explains the acid-base status change in this patient?
. Acute renal failure
. Bowel ischemia
. Loop diuretic therapy
. Opioid medication use
. Right lower lobe atelectasis
108) A 58-year-old man is admitted to the intensive care unit with an exacerbation of his chronic obstructive pulmonary disease (COPD) and respiratory failure. After several hours on noninvasive ventilation the patient develops worsening respiratory distress and requires endotracheal intubation. Which of the following is a contraindication to the use of succinylcholine for rapid-sequence intubation?
. Hyperkalemia
. COPD exacerbation
. Hepatic failure
. Hypokalemia
. Hypercalcemia
109) A 66-year-old white male comes to the physician's office for the first time because of generalized muscle weakness. His review of systems is otherwise negative. He has a past medical history of hypertension, type- 2 diabetes, hyperlipidemia, chronic renal insufficiency and ischemic cardiomyopathy. Neurological examination shows mild weakness of the lower limbs, depressed reflexes and normal sensation. Laboratory studies show: Serum sodium 134 mEq/L, Serum potassium 6.0 mEq/L, Bicarbonate 24 mEq/L, Blood urea nitrogen (BUN) 38 mg/dl, Serum creatinine 2.8 mg/dl, Calcium 8.2 mg/dl, Blood glucose 298 mg/dl. A dose of sodium polystyrene sulfonate is given. Which of the following additional interventions is most important in this patient's management?
. Review all his current medications
. Measure serum renin and aldosterone levels
. Obtain electromyography
. Obtain acetylcholine receptor antibodies
. Start oral prednisone therapy
110) A 79-year-old female presents to your office with a three-day history of nausea, diarrhea, poor oral intake and weakness. Her past medical history is significant for hypertension treated with enalapril and diltia zem. Her serum chemistry shows the following: Sodium 139 mEq/L, Potassium 7.8 mEq/L, Bicarbonate 14 mEq/L, Chloride 95 mEq/L, Glucose 155 mg/dl, BUN 80 mg/dl, Creatinine 3.1 mg/dl. EKG reveals QRS prolongation and prominent T wave peaking in precordial leads. Which of the following pharmacologic therapies plays a role in treating this patient's electrolyte condition?
. Stimulating alpha 1-adrenoreceptors
. Stimulating beta 1-adrenoreceptors
. Stimulating beta2-adrenoreceptors
. Blocking alpha 1-adrenoreceptors
. Blocking beta 1-adrenoreceptors
111) A 42-year-old male is brought to the emergency department immediately after having a prolonged seizure episode. His family describes a past medical history of grand mal seizures. He has been on phenytoin for the past 10 years, but stopped taking the drug six months ago because he had not had any seizures in the last nine years. He is otherwise healthy and had been doing well until this seizure episode. He does not use tobacco, alcohol or drugs. He is afebrile. His blood pressure is 136/88 mm Hg, respirations are 18/min and pulse is 96/min. He appears confused and lethargic. Chest auscultation is unremarkable, and his abdomen is soft and nontender. A limited neurologic examination is non-focal. His laboratory report shows: Serum sodium 140 mEq/L, Serum potassium 4.0 mEq/L, Chloride 103 mEq/L, Bicarbonate 17 mEq/L, Blood urea nitrogen (BUN) 20 mg/dl, Serum creatinine 0.8 mg/dl, Blood glucose 98 mg/dl. Chest x-ray and urinalysis are within normal limits, and a CT scan of the head is unremarkable. Which of the following is the most appropriate next step in the management of this patient's metabolic acidosis?
. Give intravenous bicarbonate
. Check serum ketones
. Check serum lipase
. Observe and repeat the labs after 2 hours
. Start dopamine
112) A 79-year-old woman presents to her primary care physician complaining of memory loss. She reports increasing difficulty balancing her check book and remembering the names of new acquaintances over the last several months. Her only other complaint is occasional urinary incontinence, which she attributes to old age. She denies headache, vision changes, rash, nausea, or vomiting. Her past medical history is significant for hypertension controlled with felodipine. On exam her vital signs are within normal limits. The heart and lung exams are also unremarkable. The cranial nerves are intact, fundoscopic exam is normal, and no tremor is observed. Her gait is slow and shuffling, there is no dysmetria, and her Folstein mini-mental status score is 24/30. CT scan of her brain is shown below. Which of the following interventions is most likely to relieve her symptoms?
. Ventriculoperitoneal shunt
. Hematoma evacuation
. Levodopa/carbidopa
. Vitamin B12 replacement
. High-dose penicillin
113) A 50-year-old male patient comes to the office because he is concerned about the marked tremors of his hands. His tremors disappear with voluntary activity and worsen with emotional stress. He finds it mild difficult to initiate movements. He does not have a family history of tremors. Physical examination reveals tremors that occur at a frequency of 3-4 cycles/sec. There is rigidity of his limb musculature. His gait and posture is minimally disturbed. His higher mental functions are intact. Which of the following is the most appropriate treatment for this patient?
. Benztropine
. Amantadine
. Selegiline
. Clozapine
. Propranolol
114) A 67-year-old male presents with a six-month history of rigidity, gait problems, tremor and slowness of movements. His condition has progressively worsened over the last few months, and he is now unable to perform his routine daily activities due to the slowness of his movements. He is not taking any medications. On examination, he is alert and conscious. His face is without expression. There is a resting tremor of his hands. He has a stooped posture and shuffling gate. There is rigidity of his limb muscles. Which of the following is the most appropriate initial treatment for this patient?
. L-dopa
. Amantadine
. Selegiline
. Clozapine
. Benztropine
115) A 58-year-old woman presents to the emergency department with severe headache and agitation. She describes her pain as right-sided and retro-orbital, and also reports blurred vision, constipation, and vomiting. Her medical history is significant for Parkinson's disease, hypothyroidism, hypertension and chronic hepatitis C. Work-up reveals that her current condition is medication-induced. Which of the following agents is most likely responsible?
. Levodopa
. Selegiline
. Bromocriptine
. Trihexyphenidyl
. Propranolol
116) A 67-year-old male comes to the office for a routine physical exam. He retired this year and wants "a clean bill of health." He has no complaints. He stopped smoking 10 years ago, but smoked for 40 years prior to that. He only takes a "water pill" for hypertension. His vital signs are normal. The physical examination reveals a bruit in his neck. His chest x-ray, EKG, and blood work have normal results. Duplex ultrasonography of his neck reveals a 70% irregular lesion at the right common carotid artery bifurcation. The left common carotid artery has a 40% lesion. What is the best next step in the management of this patient?
. Left carotid surgery
. Right carotid surgery
. Long term ASA therapy
. Temporal artery biopsy
. Heparin
117) A middle-aged woman is found wandering the streets with an abnormal gait. Police officers bring her to the hospital. She mumbles when asked for her name and age. She is not oriented to time or place. Her blood pressure is 160/100 mmHg and her heart rate is 100/min. She is afebrile. Mucous membranes are moist and the pupils are dilated and reactive to light. She moves all of her extremities, and her deep tendon reflexes are symmetric. Which of the following is the best initial treatment for this patient?
. Naloxone
. Flumazenil
. Thiamine
. Haloperidol
. Clonidine
118) A 61-year-old Caucasian male presents with ptosis, diplopia and limb weakness. These symptoms worsen in the evening and with exercise, and improve with rest. He also has fatigue, which is worse in the evening. He denies any tingling or numbness. On examination, he cannot sustain an upward gaze, and his eyelids tend to drift downward. Injection of edrophonium quickly restores power, and allows him to maintain an upward gaze. Which of the following is the best initial treatment for this patient?
. Treatment with pyridostigmine
. Treatment with edrophonium
. Treatment with atropine
. Treatment with prednisolone
. Treatment with intravenous immunoglobulins
119) A 7-year-old boy is brought to the office by his mother because, "he is always daydreaming." Sometimes, he stares for a few seconds and rolls his eyes for unknown reasons. His teacher has noted a recent decline in his school performance. He has no family history of any seizure disorder, and his psychomotor development is normal. His neurological examination is unremarkable. EEG during hyperventilation shows generalized, symmetrical 3-Hz spike-and-wave activity on a normal background. Which of the following medications is the most appropriate to treat the boy's disorder?
. Phenytoin
. Phenobarbital
. Valproic acid
. Clonazepam
. Lamotrigine
120) A 32-year-old Caucasian male comes to the emergency department due to progressive ascending paralysis, which began 18 hours ago. He initially noticed paresthesias in his lower limbs, followed by a sense of fatigue and weakness. He denies any history of headache, fever, and recent infection or illness. His blood pressure is 120/80 mm Hg, pulse is 80/min, respirations are 16/min, and temperature is 37.3°C (99.2°F). The physical examination reveals intact cranial nerves, absent deep tendon reflexes, and a normal sensory exam. Laboratory studies reveal a normal WBC count. No abnormalities are noted on CSF examination. While evaluating the patient in the hospital, he quickly deteriorates. What is the most appropriate next step in the management of this patient?
. IV immunoglobulin and plasmapheresis
. Administer botulinum antitoxin
. IV methylprednisolone
. Meticulous search for a tick
. MRI of the spine
121) A 36-year-old Caucasian male is brought to the emergency department due to weakness of his upper and lower extremities. Neurological examination reveals weakness, atrophy, fasciculations, spasticity and hyperreflexia of the involved muscles. His sensory, bowel, bladder and cognitive functions are intact. Serum creatine kinase is normal. Cerebrospinal fluid examination is within normal limits. Electromyography shows chronic partial denervation. The patient is subsequently diagnosed with amyotrophic lateral sclerosis. Which of the following has been approved for use in patients with amyotrophic lateral sclerosis?
. Riluzole
. Corticosteroids
. Intravenous immunoglobulins
. Plasmapheresis
. Donepezil
122) A 38-year-old female presents with one week of "shakiness" in her right arm, right leg weakness and unsteady gait. An MRI of her brain is shown below. Which of the following is the best treatment for this patient?
. Aspirin and simvastatin
. Glatiramer acetate
. Argatroban
. Tissue plasminogen activator
. Broad-spectrum antibiotics
123) A 62-year-old Caucasian man with hypertension and chronic kidney disease complains of poor sleep. He describes a sensation of spiders crawling over both legs about 15-20 minutes after going to bed. Sometimes he has to sit up in bed and massage his legs. His wife's sleep has deteriorated as her husband's leg movements have gotten worse. Pharmacotherapy for this patient's disease is typically directed at which of the following?
. Norepinephrine
. Dopamine
. Serotonin
. Acetylcholine
. GABA
124) A 22-year-old white obese female presents with headache for the last few weeks. Headache is worse at night and wakes her from sleep. Headache is pulsating in quality and is also associated with nausea and vomiting. She denies any weakness, sensory abnormalities or visual problems. She denies any history of trauma. She does not take any medication. Neurological examination is unremarkable. Fundoscopy shows papilledema. CT scan of head does not show any abnormality. Lumbar puncture is performed and CSF examination is normal except increased CSF pressure. Weight loss fails to control her symptoms. Which of the following is the most appropriate next step in the management of this patient?
. Treatment with mannitol
. Treatment with acetazolamide
. Treatment with corticosteroids
. Repeated lumbar punctures
. Surgery
125) A 52-year-old Caucasian male comes to the office for the evaluation of tremors. He has been having tremors in both hands and head over the past several months. He does not experience any tremors at rest, or any problem with his legs. He has a history of alcohol abuse. He is currently not on any medication. His father had essential tremors, and his paternal uncle has Parkinsonism. The physical examination reveals tremors, which increase in amplitude when he tries to reach for an object. Which of the following is the most appropriate next step in the management of this patient?
. Benztropine
. Propranolol
. Haloperidol
. Valproic acid
. Clonazepam
126) A 32-year-old Caucasian male is admitted to the hospital due to a 1-week history of progressive paralysis of his upper and lower extremities. He had a flu-like illness 3 weeks ago, followed by paresthesias in his fingertips and toes. The weakness initially started in his lower extremities. He denies any changes in bowel and bladder functions. His blood pressure (supine) is 130/70mm Hg, heart rate is 82/min, respirations are 18/min, and temperature is 36.9°C (98.5°F). Physical examination reveals bilateral facial paralysis, orthostatic hypotension, areflexia in all four extremities, and distal paresthesias. His CSF analysis shows: Total WBC 10/cmm, Protein 120 mg/dl, Glucose 70 mg/dl, Gram stains No organisms. What is the most appropriate next step in the management of this patient?
. Intravenous methyl prednisolone
. Intravenous immunoglobulin therapy
. Intravenous acyclovir therapy
. Botulinum antitoxin
. Intravenous ceftriaxone and ampicillin
127) A 59-year-old white male comes to the office for the evaluation of a brief episode of right arm and leg weakness. The episode lasted for a few minutes, and was followed by a complete recovery. He had a similar episode one month ago. He has a 30 pack-year history of cigarette smoking. He has hypercholesterolemia, which is being treated with diet and exercise. His pulse is 76/min, regular, and blood pressure is 130/80 mmHg. His laboratory test results are: Hb 14.2 g/dl, WBC 7,000/cmm, Platelets 230,000/cmm, Blood Glucose 118 mg/dl, Serum Na 138 mEq/L, Serum K 4.5 mEq/L, BUN 16 mg/dl, Serum Creatinine 1.0 mg/dl. EKG shows normal sinus rhythm. CT scan of the head is unremarkable. MRI angiography of the head and neck fails to show any abnormality. Transesophageal echocardiography (TEE) is unremarkable. Which of the following is the most appropriate next step in management?
. Treatment with aspirin
. Treatment with heparin followed by warfarin
. Treatment with clopidogrel
. Treatment with combination of aspirin and dipyridamole
. Treatment with ticlopidine
128) A 65-year-old lady comes to the office for the evaluation of her deteriorating memory. She has become increasingly forgetful over the last several months, and now appears very concerned about her memory loss. She used to pride herself for her sharp memory, but has been forgetting the most trivial things and has become "extremely inefficient." She also complains of easy fatigability, poor appetite, and frequent awakening at night. She feels worthless and has lost interest in her favorite hobby, which is gardening. On coughing or laughing, she loses urine involuntarily, and this is adding to her misery. She lives with her husband, who says that she has become very "cranky and irritable" lately. Her medical history is significant for hypercholesterolemia, for which she refuses to take medication. She is presently not on any hormonal therapy. The physical examination is completely normal. Laboratory studies are unremarkable. CT scan of the head is normal. What is the best next step in the management of this patient?
. Start hormonal replacement therapy
. Treat her with donepezil
. Start selective serotonin reuptake inhibitor
. Surgical bypass shunting
. Reassurance
129) A 27-year-old Caucasian female with multiple sclerosis (MS) comes to the office for a follow-up visit. Six months ago, she experienced monocular vision impairment and clumsiness of the right hand, and both symptoms resolved completely. Three weeks ago, she experienced incoordination, weakness and spasticity in the right extremities. She was admitted in the hospital for treatment of an acute MS flare, and rapidly improved thereafter. Physical examination currently demonstrates slight weakness of the right leg with a hyperactive knee jerk. Which of the following medications may slow the long-term progression of this patient's disease?
. High-dose corticosteroids
. Interferon-beta
. Cyclosporine
. Methotrexate
. Mitoxantrone
130) A 45-year-old man is brought to the emergency department following a serious motor vehicle collision. Despite heroic attempts to save him, the physician caring for him believes he is brain dead. Unfortunately, the patient does not have an advanced directive or durable medical power of attorney. What step is necessary in order to remove the patient from the ventilator?
. Ask his family members
. Ask the hospital ethics committee
. Confirm brain death with another physician
. Notify the county medical examiner
. No further steps are necessary
131) A 32-year-old female is crying as she approaches the office. She has had severe, unbearable pain in her face for the past five days. She describes the pain as knife-like, comes in paroxysms, occurs 10-20 times a day, and lasts a few seconds. She does not know what precipitates the attacks, but she has been unable to sleep, eat or go to work because of her symptoms. She has tried numerous pain medications, but nothing seems to relieve the pain. She denies any history of trauma, medication use or recent surgery. Vital signs are within normal limits. Physical examination is within normal limits. Which of the following agents will best benefit this patient?
. Levodopa
. Methotrexate
. Carbamazepine
. Lithium
. Morphine
132) A 23-year-old white female presents with an acute onset of headache for the last eight hours. Her headache is severe, unilateral, pulsating in quality, associated with photophobia, worsens with physical activity, and does not respond to acetaminophen or ibuprofen. This is her 6th episode of similar headache over the last 2 months, and it is her first time to seek medical attention. Her neurological examination is unremarkable. Which of the following is the most appropriate next step in the management of this patient?
. Propranolol
. Verapamil
. Ergotamine
. Rizatriptan
. Prochlorperazine
133) A 54-year-old male presents to the office with several months history of hand tremors that are unresponsive to over-the-counter medication. The tremors always become worse when he is in public places. Sometimes, the hand tremors are so bad that he is unable to grasp. He denies trauma, fever, loss of muscle function or any prior stroke. His past medical history is significant for vague abdominal pains, the cause of which was never found. He is given some medication. A month later, he comes back and says, "The tremor is gone, but now I have colicky abdominal pain, confusion, headaches, hallucinations and dizziness." What is the drug that caused this patient's new symptoms?
. Alcohol
. Propranolol
. Primidone
. Diazepam
. Lithium
134) A 44-year-old male has been abusing drugs for many years. Recently he started using the agent MPTP and he now presents to you in the ER. He says that he is not feeling well and wonders what is going on. He says he has a persistent tremor in his fingers and his body has become rigid. At times has had difficulty in walking. The CT scan of his brain is normal and all the blood work is within normal limits. You decide to treat him. Two years later you see him and he tells you that his symptoms have improved. He also noticed that he had a significant relief from the recurrent and persistent upper respiratory tract viral infections that he was so prone to. The agent you gave him was?
. Bromocriptine
. Levodopa-Carbidopa
. Amantadine
. Benztropine
. Acyclovir
135) A 5-year-old boy is brought to the office by his frustrated mother because, "he just can't seem to concentrate!" He has episodes that are characterized by frequent blinking and indifference to his surroundings. He does not remember anything after an episode. An episode does not last long, but he usually has 50-80 "daydreaming accidents" daily. His school performance has deteriorated lately, and his teacher has complained about his decreased concentration. Which of the following is the drug of choice for this patient's condition?
. Phenytoin
. Carbamazepine
. Ethosuximide
. Phenobarbital
. Lamotrigine
136) A 27-year-old white female comes to the office and complains of ptosis, diplopia and difficulty in chewing. Her symptoms worsen in the evening. She is asymptomatic when she wakes up in the morning. She denies any sensory complaints or limb weakness. The Tensilon test is positive. Serological testing shows positive acetylcholine receptor antibodies. Which of the following is the best treatment to induce remission and provide long-term benefits in this patient?
. Pyridostigmine
. Prednisolone
. Intravenous immunoglobulins
. Plasmapheresis
. Thymectomy
137) A 62-year-old male is brought to the emergency department with a chief complaint of weakness. He says that he was walking his dog 1 hour ago when he began to limp and noticed some weakness in his left arm. His past medical history is significant for hypertension treated with hydrochlorothiazide, and type II diabetes mellitus managed with metformin. He denies nausea, vomiting, chest pain, fever or chills. He does note occasional palpitations and tension headaches at baseline. On physical examination, his blood pressure is 170/95 mmHg and his heart rate is 76/min and regular. His blood glucose level is 190 mg/dl and his HbA1c is 7.6%. The neurological examination is significant for profound left-sided weakness and an up going Babinski reflex on the left. Non-contrast head CT is negative for any intracranial bleed. Which of the following is most likely to affect his chance of neurological recovery?
. Insulin for tight glucose control
. Labetalol
. Aspirin
. Fibrinolytic
. Heparin
138) A 62-year-old female is brought in by EMS due to a severe right-sided headache, nausea and eye pain. She was fixing a light bulb, when she suddenly felt pain in her right eye. She decided to rest, but the eye pain only got worse. In the next few minutes, she developed loss of vision, photophobia and redness in the same eye. She took medications to relieve the accompanying headache, but the eye pain persisted. She denies any trauma. Her past medical history is significant for diabetes and hypertension. She appears to be in intense pain with bouts of nausea. Her right eye is red, with conjunctival flushing and visual acuity of 20/200. Her right pupil is mid-dilated and non-reactive to light. The same eye feels hard on palpation. The one treatment that should be avoided in this patient is:
. Mannitol
. Acetazolamide
. Pilocarpine
. Timolol
. Atropine
139) A 65-year-old male comes to the emergency department because of a sudden, painless loss of vision in his right eye that occured a half an hour ago. Five hours ago, he experienced a similar but transient loss of vision in the same eye, which lasted for five minutes. He has hypertension, diabetes, hypercholesterolemia, and peripheral vascular disease. He had an anterior wall myocardial infarction six years ago. His medications include glyburide, captopril, atenolol, simvastatin and aspirin. His temperature is 36.7°C (98.0°F), respirations are 16/min, pulse is 88/min, and blood pressure is 146/88 mmHg. Examination of the right eye reveals visual acuity of 20/60 and subtle retinal whitening. A right carotid bruit is heard. Which of the following is the most appropriate next step in the management of this patient?
. Acetazolamide IV
. Ocular massage and high flow oxygen
. Administer systemic steroids
. Instillation of topical beta blocker
. Administer thrombolytics
140) A 33-year-old man presents with a 1-day history of localized, small swelling along the margin of the upper eyelid. He feels pain, which does not seem to come from the conjunctival surface. He has a 10-pack year smoking history. He occasionally drinks alcohol. He is sexually active, and does not use condoms regularly. He is worried about the swelling. What is the best next step in the management of this patient?
. Use warm compresses
. Incision and drainage
. Incision and curettage
. Take biopsy of the lesion
. Oral Penicillin
141) A 72-year-old man presents with right eye pain for 1 day. The patient has a history of diabetes, hypertension, and “some type of eye problem.” He does not recall the name of his eye problem or the name of his ophthalmic medication. However, he does remember that the eye drop has a yellow cap. Which class of ophthalmic medication is the patient taking?
. Antibiotic
. β-Blocker
. Mydriatic/cycloplegic agent
. Miotic
. Anesthetic
142) A 45-year-old woman presents with right eye pain and redness for 1 day. She has photophobia and watery discharge from the eye. She does not wear glasses or contact lenses and has no prior eye problems. On examination, the patient’s visual acuity is 20/20 in the left eye and 20/70 in the right eye. She has conjunctival injection around the cornea and clear watery discharge. On slit-lamp examination, the lids, lashes, and anterior chamber are normal. When fluorescein is applied, a branching, white-colored epithelial defect is seen. The remainder of the head examination is normal and the patient has no cutaneous lesions. Which of the following is the most appropriate treatment for this patient?
. Admission for intravenous (IV) antibiotics
. Admission for IV antiviral agents
. Topical steroids
. Topical antiviral medication
. Immediate ophthalmology consultation
143) A 21-year-old man presents to the ED with a red eye. The patient complains of rhinorrhea and a nonproductive cough but has no eye pain or discharge. He also has no associated ecchymosis, bony tenderness of the orbit, or pain on extraocular eye movement. His vision is normal, extraocular movements are intact, and intraocular pressure (IOP) is 12. A picture of his eye is shown below. What is the most appropriate management of this condition?
. Call ophthalmology immediately
. Administer 1% atropine
. Elevate patient’s head
. Administer ophthalmic timolol
. Reassurance only
144) A 28-year-old mechanic with no past medical history presents to the ED after a small amount of battery acid was splashed in his right eye. He is complaining of extreme pain and tearing from his eye. Which of the following is the most appropriate next step in management?
. Call ophthalmology now
. Check visual acuity
. Check the pH of the tears
. Irrigate with normal saline
. Apply erythromycin ointment
145) A 33-year-old white female complains of repeated episodes of fever, malaise, chills, breathlessness and dry cough over the past 6 months. She says that each episode starts suddenly and lasts for several days. She otherwise has no significant past medical history and does not take any medications. There is no family history of lung disorders. She does not smoke cigarettes or drink alcohol. She has never been abroad, and has had no sick contacts. She breeds budgerigars (a small Australian parrot) as a hobby. Chest x-ray shows diffuse generalized haziness in both lower lung fields. Pulmonary function tests reveal reduced lung volumes and an FEV1/FVC ratio of 87%. PPD test is negative. Serology shows antibodies to budgerigar antigens. What is the best treatment for this disease?
. Inhaled beclomethasone
. Oral prednisolone
. Inhaled cromolyn
. Avoid exposure to birds
. Reassurance
146) A 44-year-old male is involved in a serious motor vehicle accident. He is admitted to the intensive care unit with multiple fractures and internal bleeding, and is placed on mechanical ventilation. Over the next two days, his oxygen saturation repeatedly drops below 90%, requiring gradual increases in the FiO2. On day three of his admission, the patient's chest x-ray, which was initially clear, shows bilateral fluffy infiltrates. He has no prior medical problems and does not take any medications. Skin examination reveals no rashes. Lab studies show: Hemoglobin 10.1 g/L, Platelets 160,000/mm3, Leukocyte count 13,500/mm3. Sputum cultures are negative. The ventilator settings are: FiO2 90%, Tidal volume 400 ml (body weight 60 kg), Respiratory rate 12/min, PEEP 5cmH2O. The patient's oxygenation continues to drop below 90%. Which of the following interventions would be best for improving the patient's oxygenation?
. Increase the respiratory rate
. Increase the tidal volume
. Increase the PEEP
. Increase the intravenous fluids
. Administer packed red blood cells
147) A 65-year-old white man comes to the Emergency Room complaining of headache, insomnia, palpitations, and vomiting. His past medical history is significant for chronic obstructive pulmonary disease (COPD) treated with theophylline, ipratropium, and occasional albuterol. He had a puncture wound of the foot one week ago, and it is being treated effectively with amoxicillin-clavulanate and ciprofloxacin. For the past three days, his shortness of breath is worsening, and his primary care physician gave oral prednisone. His blood pressure is 150/80 mmHg and heart rate is 105/min with frequent ectopic beats. You suspect that drug toxicity may be responsible for this patient's complaints. Which of the following drugs is most likely responsible for his condition?
. Theophylline
. Ipratropium
. Albuterol
. Ciprofloxacin
. Steroids
148) A 67-year-old woman with a past medical history significant for hypertension, hypercholesterolemia, and type 2 diabetes calls 911 for severe shortness of breath. Her symptoms started 2 hours ago with chest pain and progressed rapidly to orthopnea and shortness of breath. Her blood pressure is 170/ 100 mmHg and heart rate is 120/min and regular. A third heart sound is present. Bilateral crackles are heard on chest auscultation. Her oxygen saturation is 78% with 40% inspired oxygen. She is intubated in the field by paramedics for progressive respiratory failure and is treated with nitrates and diuretics. After the initial resuscitation, breath sounds on the left side are markedly decreased. Which of the following is most likely to restore breath sounds to this hemithorax?
. Left-sided chest tube
. Left-sided needle thoracostomy
. Pericardiocentesis
. Endotracheal tube withdrawal
. Tidal volume increase
149) A 55-year-old chronic smoker comes to you because of worsening shortness of breath. He says that he has had lung problems due to smoking for the past five years. He has hypertension controlled with hydrochlorothiazide, and type 2 diabetes mellitus, controlled with diet. He has smoked two packs of cigarettes daily for 25 years. Physical examination shows bilateral decrease in breath sounds with prolonged expiration and wheezing in both the lung fields. He is hypoxic on room air. An x-ray film of the chest shows hyperinflation of both lung fields. His pulmonary function tests showed moderate obstructive disease, with very little bronchodilator response. The patient is started on bronchodilators. He is being considered for home oxygen therapy. Which of the following is a criterion for initiating home oxygen in such patients?
. Worsening of shortness of breath
. Patients with SaO2 less than 88% on room air
. Patients with PaO2 1ess than 65 mmHg on room air
. Patients with SaO2 less than 92% on room air
. Development of pulmonary osteoarthropathy
150) A 65-year-old white male who has a history of chronic obstructive pulmonary disease, congestive heart failure, and coronary artery disease presented with a three-day history of worsening shortness of breath, cough with yellowish expectoration, and fever. He is not on steroids and does not use oxygen at home. He takes ipratropium, albuterol, aspirin, digoxin, furosemide, metoprolol, and lisinopril. He lives at home with his wife. His temperature is 39.4°C (103°F), blood pressure is 110/70 mmHg, pulse is 110/min, and respirations are 24/min. He is saturating 88% on room air. He was started on 3-liters of oxygen to keep the saturations above 92%. The chest x-ray of the patient is shown below. What is the most appropriate next step in the management of this patient?
. Admit the patient and give ampicillin
. Outpatient trimethoprim-sulfamethoxazole
. Admit the patient and start levofloxac
. Admit the patient and start ciprofloxacin
. Admit the patient and start vancomycin
151) A 76-year-old male nursing home resident is hospitalized with confusion, fever and decreased oral intake. His past medical history is significant for type 2 diabetes mellitus, hypertension, osteoarthritis and gout. Blood cultures are positive for E coli. Despite antibiotic therapy, he slips into respiratory failure. He is intubated and placed on mechanical ventilation with an FiO2 of 70%, tidal volume of 500 ml and respiratory rate of 14/min. His current arterial blood gases are: pH 7.45, pO2 59 mmHg, pCO2 30 mmHg, HCO3 21 mEq/L. Which of the following is the best next step in managing this patient?
. Increase tidal volume
. Increase respiratory rate
. Increase the fraction of inspired oxygen
. Add positive end-expiratory pressure
. Decrease the fraction of inspired oxygen
152) A 45-year-old male with mild persistent asthma comes to you for a routine checkup. He is taking a low-dose beclomethasone inhaler daily and inhaled albuterol, as needed, for the last year. His past medical history is otherwise not significant. His family history is significant for diabetes mellitus type 2, hypertension and obesity. His blood pressure is 136/90 mmHg and his heart rate is 80/min. His BMI is 26 kg/m2. Which of the following is the most likely complication of chronic beclomethasone use in this patient?
. Cushing's syndrome
. Thrush
. Osteoporosis
. Adrenal suppression
. Purpura
153) A 56-year-old morbidly obese man twisted his ankle five days ago and has been bed-ridden since. Today he presents to the emergency department with acute-onset shortness of breath and chest tightness. His past medical history is significant for diabetes mellitus, hypertension, right knee osteoarthritis, and a deep venous thrombosis. His current medications are lisinopril, metoprolol, and metformin. His blood pressure is 110/60 mmHg and his heart rate is 110/min. A CT scan of the chest with contrast is consistent with pulmonary thromboembolism. Which of the following is the best treatment for this patient?
. Start heparin now, and in 5-6 days stop heparin and start warfarin
. Start both heparin and warfarin now, and stop heparin in 1-2 days
. Start heparin and warfarin now, and stop heparin in 5-6 days
. Start heparin now and continue for 3-4 weeks
. Start warfarin now and continue for at least 6 months
154) A 35-year old male presents to your office complaining of nocturnal wheezing and chest tightness for the past three months. He has also noticed new hoarseness, particularly in the morning. He has no history of bronchial asthma, hypertension or diabetes. He is a non-smoker but occasionally drinks alcohol. On examination, he is an obese male in no acute distress. His pulse is 84/min, blood pressure is 130/80 mmHg, and respirations are 16/min. His chest is clear to auscultation and percussion. Laryngoscopy reveals a red and inflamed posterior pharynx and larynx. Which of the following pharmacotherapies would be most helpful for this patient?
. Bedtime fluticasone inhaler
. Bedtime albuterol inhaler
. Oral theophylline
. Oral omeprazole
. Oral prednisone
155) A 38-year-old female comes to your office with a recent episode of hemoptysis. The symptoms started one week ago with malaise, throat pain and dry cough. The cough progressed becoming productive of yellowish sputum. She started noticing speckles of red blood in her sputum as of yesterday. Her past medical history is significant for peptic ulcer disease. She has a 5 pack-year smoking history, but she quit 10 years ago. Her temperature is 98°F (36.7°C), blood pressure is 130/80 mmHg and heart rate is 87/min. Physical examination reveals scattered bilateral wheezes. Chest X-ray shows clear lung fields. Which of the following is the most appropriate next step in management?
. CT scan of the chest
. Pulmonary function tests
. Sputum Gram stain and culture
. Observation
. Sputum cytology
156) A 34-year-old male presents to his physician's office for a routine health maintenance examination. He has a five-year history of bronchial asthma for which he uses an albuterol inhaler. He says that he uses the inhaler an average of two times per week during the day. In addition, he states that his asthma symptoms wake him from sleep approximately 3-4 times per month. He does not use tobacco, alcohol or illicit drugs. His family history is significant for asthma in his grandfather. Physical examination is unremarkable. Which of the following is the most appropriate next step in his management?
. Add long-acting beta-2 agonist inhaler
. Add inhaled corticosteroids
. Add oral theophylline
. Add oral prednisone
. Continue current medical regimen
157) A 19-year woman presents to her physician's office for a routine health maintenance examination. She notes experiencing occasional wheezing and breathlessness following aerobic exercise, but has no symptoms at other times of the day or night. Her medical history is otherwise significant for allergic rhinitis and acne, for which she uses topical benzoyl peroxide and erythromycin cream. She does not use tobacco, alcohol or illicit drugs. On physical examination, her vital signs are within normal limits and chest auscultation is unremarkable. Which of the following is the most appropriate next step in her management?
. Beta-adrenergic agonists before exercise
. Daily oral steroids
. Daily steroid inhalers
. Daily oral theophylline
. Lpratropium inhalers before exercise
158) A 57-year-old male was involved in a motor vehicle accident and was immediately brought to the Emergency Room. He suffered minor head and neck injury, but was found to have a broken tibia. He underwent repair of the tibial fracture the following day. Five days later, he suddenly develops tachycardia and is tachypneic. He complains of increasing chest discomfort and has a syncopal episode. His blood pressure is 80/55 mmHg, pulse is 130/min, and respirations are 24/min. Pulse oximetry showed 82% on room air. Stat echocardiogram reveals obstruction in the main pulmonary artery. Which of the following choices is the most appropriate course of action?
. Heparin infusion
. Fibrinolytic therapy
. Embolectomy
. Inferior vena cava filter
. A CT scan of the lung
159) A 68-year-old female comes to the Emergency Room with increased shortness of breath and cough for 12 hours. She has a history of chronic obstructive pulmonary disease for the past 14 years and has been oxygen dependent for two years. Twelve hours ago, she felt unable to get her breath. She took two extra nebulizer treatments and used her as-needed inhaler with no relief of symptoms. She denies any fever, chills, sweating, nausea or vomiting, and hemoptysis. She rarely uses alcohol, but had smoked two packs/day for 45 years. She quit smoking about six years ago. Her medications include ipratropium, albuterol, multivitamins, and oxygen 2 liters/min by nasal cannula. She is a thin, frail female in moderate distress. Her temperature is 36.7°C (98°F), blood pressure is 110/65 mmHg, pulse is 110/min, and respirations are 28/min. Pulse oximetry shows 84% on 2 liters oxygen by nasal cannula. The patient is given oxygen, methyl prednisolone, aerosolized ipratropium, and albuterol; however, the patient remains dyspneic and bedridden. ABGs are ordered and the results show: pH 7.32, PCO2 60 mmHg, and PO2 52 mmHg on 4-liters O2. What is the next best step in the management of this patient?
. Increase oxygen by nasal cannula
. Decrease oxygen
. Intubate and mechanically ventilate the patient
. Noninvasive positive pressure ventilation
. Aminophylline
160) A 68-year-old male is hospitalized following an intracerebral hemorrhage. On hospital day 3, he develops redness, swelling, and tenderness of the left calf. There is no fever. Compression ultrasonography reveals a thrombus in the popliteal vein. Which of the following is the most appropriate next step in the management of this patient?
. Anticoagulation with heparin
. Thrombolytic therapy
. Placement of an inferior vena cava filter
. No treatment for his deep vein thrombosis
. Compression stockings
161) A 66-year-old man complains of exertional dyspnea that has progressed over the last two years. As a result, he has had to limit his physical activities to avoid becoming short of breath. He denies any significant chest pain, but does note having a recurrent cough productive of whitish sputum. His past medical history is significant for hypertension controlled with hydrochlorothiazide. He has a 40 pack-year smoking history. His family history is significant for a stroke in his mother. His blood pressure is 160/90 mmHg and his heart rate is 80/min. Physical examination reveals a mildly overweight patient in no acute distress. His anteroposterior chest diameter is markedly increased. Breath sounds are decreased bilaterally with scattered expiratory wheezes. Which of the following agents is most likely to reduce this patient's symptoms?
. Alpha-adrenergic blocker
. Beta-adrenergic blocker
. Muscarinic antagonist
. Dopamine agonist
. Alpha 2-adrenergic agonist
162) A 64-year-old male presents to the physician's office with increasing shortness of breath. He denies orthopnoea, paroxysmal nocturnal dyspnea, or chest pain. He was hospitalized for pneumonia four years ago but has otherwise been healthy. He has smoked one pack of cigarettes daily for the past 30 years. He does not take any medications. His temperature is 37.2°C (98.9°F), blood pressure is 124/76 mm Hg, pulse is 82/min and respirations are 16/min. Pulse oximetry reveals an oxygen saturation of 88%. On chest auscultation, breath sounds are diminished throughout, and the expiratory phase is prolonged. Heart sounds are regular and there are no murmurs or gallops. There is no peripheral edema or jugular venous distention. Laboratory studies show a hematocrit of 56% and WBC count of 6,700/mm3. Which of the following interventions will have the maximum impact on this patient's survival?
. Maintenance oral steroids
. Prophylactic antibiotics to reduce exacerbations
. Influenza and pneumococcal vaccinations
. Long-term supplemental oxygen therapy
. Long-term beta-blocker therapy
163) A 47-year-old male who has just returned from a cruise to the Bahamas presents to the hospital with a high-grade fever, productive cough and shortness of breath for the past two days. He also complains of frequent vomiting and diarrhea. He has no prior medical problems, does not take any medications, and denies using tobacco, alcohol or drugs. His temperature is 40°C (104°F), blood pressure is 110/65 mm Hg, pulse is 80/min, and respirations are 18/min. Lung examination reveals right lower lobe crackles. The abdomen is soft and non-tender. Chest x-ray demonstrates right lower lobe consolidation. Sputum Gram stain contains many neutrophils with no organisms. Which of the following antibiotics must be given to this patient?
. Vancomycin
. Clindamycin
. Fluconazole
. Amoxicillin
. Azithromycin
164) A 36-year-old female presented to the emergency room with fever and a productive cough. The physician on call suspected community acquired pneumonia and prescribed azithromycin for 5 days. After 5 days of treatment, she comes to your office with no improvement of her symptoms and complains of worsening foul smelling sputum. Further inquiry reveals that she had undergone an upper GI endoscopy 8 days ago for a long history of heartburn and suspected peptic ulcer disease. She also reports a history of manic-type bipolar disorder. In your office she has a blood pressure of 130/80 mm Hg, her pulse is 108/min, temperature is 38.7°C (101.6°F) and respirations are 26/min. Chest x-ray showed a right upper lobe infiltrate. Which of the following additional therapies would be most helpful for this patient's condition?
. Doxycycline
. Ciprofloxacin
. Trimethoprim-sulfamethoxazole
. Clindamycin
. Gentamicin+ ampicillin
165) A 45-year-old male comes to the emergency room complaining of shortness of breath that began 3 hours ago. He also has a nonproductive cough, a low-grade fever and right-sided chest pain that worsen with inspiration. He denies coughing up blood, wheezing, palpitations, leg pain or swelling of his lower extremities. He recently returned home from a trip to Singapore. He has a history of hypertension and diabetes mellitus. His takes fosinopril, metformin and glyburide. He doesn't use tobacco, alcohol or illicit drugs. His temperature is 98.0°F (36.6°C), blood pressure is 115/70 mmHg, pulse is 128/min, respirations are 32/min, and O2 saturation is 84% on room air. Physical examination shows a slightly obese man in acute respiratory distress. He is alert and cooperative without any cyanosis or jaundice. He has a slightly displaced apical impulse with a S4. Chest-x ray shows mild cardiomegaly with no infiltrates. EKG shows sinus tachycardia and left ventricular hypertrophy without ST-T wave changes. His arterial blood gas analysis shows pH 7.52, pCO2 30, pO2 55, HC03 22. What is the next best step in the management of this patient?
. Pulmonary angiogram
. Lower extremity venous ultrasound
. Spiral CT scans of the chest
. Initiate heparin weight-based protocol
. Broad spectrum antibiotics
166) A 65-year-old male with oxygen-dependent chronic obstructive pulmonary disease, chronic atrial fibrillation, and depression comes into the Emergency Room, with symptoms of increased dyspnea and worsening cough pattern. His recent history had been significant for a gradual worsening of his baseline lung disease over the past month, which had been treated by his outpatient doctor with increased frequency of inhaled beta-agonist and azithromycin. This morning he had a severe shortness of breath that was unresponsive to "stacked" home nebulizer treatments. The ER physician notes that the patient is in moderate severe respiratory distress. His temperature is 37.2°C (99°F), blood pressure is 150/90 mmHg, pulse is 110/min, and respirations are 28/min. Accessory muscle use was noted. Lung exam shows diffuse rhonchi and wheezing. A pulse oximetry revealed an oxygen saturation of 80% on room air. His chest x-ray showed no new infiltrates. His WBC count is 7,000/cmm with normal differential. The ER physician had given nebulization, and the patient is on 5-liters of oxygen. Which of the following should also be considered in this patient?
. Gatifloxacin
. Methylprednisolone
. N-acetylcysteine
. Clarithromycin
. Aminophylline
167) A 32-year-old man presents to the emergency department with pain and swelling in the right leg. He was recently hospitalized for a right lower extremity deep venous thrombosis and discharged on warfarin. Today his INR is 1.12. Ultrasound reveals a right popliteal vein thrombosis extending into the deep femoral vein. What is the best initial management step for this patient?
. Increase warfarin dose for goal INR > 2.0
. Start intravenous unfractionated heparin
. Initiate thrombolytic therapy
. Place inferior vena cava filter
. Discontinue warfarin and reassure
168) A 50-year-old female comes into your office complaining of four-month history of dry cough. She denies dyspnea or hemoptysis. Past medical history is significant for hypertension, diabetes, and gout. Her current medications include enalapril, metformin, hydrochlorothia zide, and allopurinol. She does not smoke or consume alcohol. Her vital signs are BP 130/80 mmHg, HR 80/min, T 36.7°C (98°F) and RR 16/min. Physical examination is within normal limits. The best next step in the management of this patient is:
. Chest radiograph
. Reevaluation of drug therapy
. Pulmonary function testing
. Barium esophagography
. Bronchoscopy
169) A 45-year-old white man presents to your office complaining of daytime somnolence, increased fatigability, and morning headaches. He says that his wife complains that he snores loudly. He is not taking any medications and does not smoke or consume alcohol. His blood pressure is 145/90 mmHg and heart rate is 90/min. Physical examination reveals a well-nourished man with BMI = 31 kg/m2. What is the next best step in the management of this patient?
. A benzodiazepine to improve sleep
. Advise to sleep in supine position
. Weight reduction
. A thiazide diuretic for hypertension
. Tracheostomy
170) A 43-year-old previously healthy male is hospitalized for acute pancreatitis. On day 3 of his admission, he develops respiratory distress. He is transferred to the intensive care unit and intubated. His initial ventilator settings are: FiO2 0.8 (80%), Respiratory rate 10/min, Tidal volume 500 ml, PEEP 5 mm H20. Ten minutes after being intubated, the man's blood pressure is 110/70 mmHg and his heart rate is 90/min. His arterial blood gases are: pH 7.42, pO2 105 mmHg, pCO2 37 mmHg. Which of the following is the best next step in the management of this patient?
. Decrease the positive end expiratory pressure (PEEP)
. Decrease the tidal volume
. Decrease the FiO2
. Decrease the respiratory rate
. Increase the respiratory rate
171) A 35-year-old male was involved in a motor vehicle injury and suffered serious chest trauma. A chest tube was placed for a hemothorax. 800 cc of blood was evacuated from the chest, and, after ten days, the patient was discharged home. He returns again with a low-grade fever and dyspnea. CT scan reveals a complex loculated effusion with a thick surrounding peel. A chest tube is placed; however, after 24 hours, there is little drainage, and the patient continues to have a low-grade fever. What is the next step in his management?
. Surgery
. Streptokinase into the chest tube
. Increase the dose of IV antibiotics
. Place a second chest tube
. Pulmonary consult for bronchoscopy
172) A 40-year-old white male develops a proximal deep vein thrombosis in the left lower extremity. Detailed history, examination and lab testing fail to reveal any obvious cause or risk factor for his deep venous thrombosis. Idiopathic deep vein thrombosis is diagnosed and the patient starts a 6-month course of warfarin anticoagulation. What is the goal INR therapeutic range in this patient?
. 1.0 to 1.5
.1.5to2.0
. 2.0 to 3.0
. 2.5 to 3.5
. Greater than 4
173) An obese 56-year-old female presents to her physician concerned that her asthma is worsening. She describes night-time cough and wheezing that have been increasing over recent months. She also reports feeling tired each morning because she works late hours and has no time to relax after dinner. On review of systems, the patient denies dyspnea on exertion, but acknowledges that her throat has been sore lately. Her past medical history is significant for bronchial asthma, type2 diabetes and hypertension. Her medications include an albuterol inhaler which she uses occasionally, lisinopril and aspirin. Her vital signs are within normal limits, and there are no abnormalities on physical exam. Which of the following is the most appropriate next step in the management of this patient?
. Discontinue lisinopril
. Discontinue aspirin
. Add inhaled fluticasone
. Add pantoprazole
. Add salmeterol
174) A 60-year-old Caucasian female comes to the physician because of progressive shortness of breath for the past month. She has had a 13.6 kg (30 lb) weight loss during this period. She has a history of hypertension and hypothyroidism and takes metoprolol and levothyroxine. She has smoked two packs of cigarettes daily for 35 years. Her temperature is 36.7°C (98°F), blood pressure is 130/70 mmHg, pulse is 80/min, and respirations are 20/min. On examination, decreased breath sounds and dullness to percussion are noted on the right, middle and lower lobes of the lung. Her chest x-ray is shown below. Which of the following is the most appropriate next step in the management?
. Bronchoscopy
. Mediastinoscopy
. Thoracentesis
. Video-assisted thoracoscopy
. Open pleural biopsy
175) A 46-year-old male is hospitalized with severe acute pancreatitis. Because of progressive respiratory difficulty, he is intubated and placed on mechanical ventilation. His weight is 70 kg (152 lb), blood pressure is 110/70 mmHg, and heart rate is 90/min. Chest x-ray shows patchy opacities bilaterally, consistent with pulmonary edema. His current ventilator settings are: assist control mode, respiratory rate of 12/min, tidal volume of 450 ml, FiO2 of 40%, and positive end-expiratory pressure (PEEP) of 5 cm H20. His blood gas readings are: pH 7.51, pCO2 22mmHg, pO2 121mmHg. Which of the following is the best next step in managing this patient?
. Bronchodilator therapy
. Decrease FiO2
. Decrease respiratory rate
. Increase tidal volume
. Incentive spirometry
176) A 69-year-old male presents to the ER with severe shortness of breath for the past 12 hours. He reports that he has used his albuterol inhaler many times without relief. On review of systems he notes having had a mild fever yesterday that resolved with acetaminophen. The patient quit smoking 2 years ago but has a 100 pack year smoking history. On physical examination, his blood pressure is 150/90 mmHg and heart rate is 110/min. The patient is using accessory muscles of respiration. Expirations are prolonged, and there are wheezes bilaterally. Arterial blood gas (ABG) analysis reveals the following: pH 7.36, pO2 64 mmHg, pCO2 57 mmHg, HCO3 32 mEq/L. The patient is treated with ipratropium and albuterol nebulizers and moxifloxacin. Which of the following additional therapies is most likely to benefit this patient?. Corticosteroids
. Corticosteroids
. Loop diuretic
. Mucolytic agents
. Aminophylline
. Diltiazem
177) A 28-year-old male presents to the physician's office for a routine health maintenance examination. He has a two-year history of bronchial asthma for which he uses an albuterol inhaler. He states that he experiences asthma symptoms an average of 2 times per week, for which his albuterol inhaler provides relief. He has not had nighttime awakenings over the past month. He does not use tobacco, alcohol or illicit drugs. His family history is significant for asthma in his grandfather. Physical examination is unremarkable. Which of the following is the most appropriate next step in his management?
. Add long-acting beta-2 agonist inhaler
. Add inhaled corticosteroids
. Add oral theophylline
. Add oral prednisone
. Continue current medical regimen
178) A 3-year-old boy presents to the emergency department with a fever and difficulty breathing. He is the product of a normal pregnancy and has been healthy since birth. His immunizations are up to date. This morning he appeared to be in his usual state of health and was dropped off at day care by his father. Later on, his teacher noticed that he had suddenly become fussy and flushed and could not be consoled with toys, rocking, or hearing a story. He also felt warm to the touch and was drooling more than usual. When she took his temperature, it was 39°C (102.2°F). His parents were contacted immediately, and the patient was brought to the emergency department. He appears toxic and anxious, and has loud labored breathing. He is sitting upright, bracing himself on his arms, with his neck hyperextended and mouth open. His temperature is 40°C (104°F), respiratory rate is 50/min, pulse is 140/min, blood pressure is 102/62 mm Hg, and oxygen saturation is 100% on room air. Lateral x-ray of the neck is shown in the image. Laryngoscopy reveals a large cherryred epiglottitis. What is the most appropriate next step in management?
Antibiotic therapy
Corticosteroids
Nasotracheal intubation
Observation
Tracheostomy
179) A 35-year-old homeless man presents to the emergency department with chief complaints of a cough and fever. He is intoxicated. He admits to drinking about a fifth of vodka every day and confirms a history of delirium tremens and blackouts. X-ray of the chest is significant for an air-fluid level in the superior segment of the right lower lobe. Which of the following is the most appropriate first-line agent for treating this patient’s condition?
Azithromycin
Clindamycin
Isoniazid
Moxifloxacin
Piperacillin-tazobactam
180) An agitated and nervous 24-year-old woman has had severe wheezing and shortness of breath for 2 days. After receiving oxygen, steroids, and salbutamol (Ventolin) in the emergency room, her breathing improves. She is still wheezing and now feels tremulous and anxious with a pulse of 110/min and respirations 30/min. Arterial blood gases on oxygen reveal a pH of 7.40, PO2 340 mm Hg, PCO2 40 mm Hg, and bicarbonate of 24 mEq/L. She is hospitalized for further treatment. Which of the following treatments or medications should be avoided in her?
Theophylline
Sedatives
Corticosteroids
Sympathomimetic amines
Intravenous (IV) fluids
181) A 29-year-old woman has a long history of mild asthma. She now has a flare and experiences recurrent episodes of bronchial obstruction, fever, malaise, and expectoration of brownish mucous plug. On examination, there is bilateral wheezing. Infection is suspected and a CXR reveals upper lobe pulmonary infiltrates. The eosinophil count is 2000/mL, and serum precipitating antibodies to Aspergillus are positive. Which of the following is the most appropriate next step in management?
Antihelminthic therapy
A short course of systemic glucocorticoid therapy
Desensitization treatment
High-dose glucocorticoids by puffer
Long-term systemic glucocorticoid therapy
182) A 31-year-old African American man presents with dyspnea on exertion. He also has fever and red tender rash on his shins. Physical examination reveals fine inspiratory crackles in both lower lung lobes and tender erythematous nodules on his legs. CXR shows bilateral hilar adenopathy and reticulonodular changes in both lungs. Transbronchial biopsy reveals noncaseating granulomas. Which of the following is the most appropriate next step in management?
Aspirin
Isoniazid (INH) and streptomycin
Steroids
Nitrogen mustard
No therapy
183) A 53-year-old man with a long respiratory his- tory is admitted to the hospital because of increasing shortness of breath and sputum production. He is started on antibiotics and inhaled bronchodilators and anticholinergic agents. The next day he is found in his room confused and sleepy. A PCO2 determination reveals severe hypercarbia (PCO2 70 mmHg). Which of the following explanations regarding his elevated PCO2 is correct?
Occurs only with CO2 inhalation
Does not occur in obstructive lung disease
Does not occur in restrictive lung disease
May worsen with oxygen administration
Occurs with chronic hypocapnia
184) A 30-year-old man presents to the resuscitation bay with gunshot wounds in the anterior and posterior left chest. Although in distress and dyspneic, the patient is cooperative. He has a patent airway and is moving all extremities. His pulse is 120/min, blood pressure is 120/90 mm Hg, and respiratory rate is 30/min. He has bounding distal pulses, and no other injuries are identified on secondary examination. X-ray of the chest reveals fluid in the pleural space, and a left chest tube thoracostomy yields 600 mL of bright red fluid. Over the next hour 750 mL of blood is collected. What is the most appropriate next step in management?
Autotransfuse with the collected blood and continue to observe closely
Insert another chest tube
Left thoracotomy
Remove the chest tube and suture the inci- sion closed
Thoracentesis
185) A 22-year-old woman with mild persistent asthma comes to the primary care clinic after an emergency department visit 2 days ago for an acute asthma exacerbation. She notes an in- crease in frequency of wheezing and shortness of breath for the past 4 months, with daily symptoms, and has been symptomatic for at least 2 nights per week. She has also had three emergency department visits during the same period. Her current asthma medications include montelukast (leukotriene inhibitor) daily and an albuterol inhaler as needed. The patient’s peak flow is 75% of predicted. Which of the following is the most appropriate next step in management?
Add a long-acting inhaled β-adrenergic agonist and low-dose inhaled steroid to the regimen
Add systemic steroids to the regimen
Admit to the hospital for further pulmonary work-up
Discontinue the leukotriene inhibitor and change the regimen to daily low-dose inhaled steroids
Start cromolyn sodium
186) A 30-year-old man has episodes of wheezing and shortness of breath two to three times per week. Approximately every 2 weeks he awakens at night due to cough and difficulties breathing. He reports having similar symptoms since he was a child, but believes that they are worsening somewhat now. His symptoms are worsened by cold air and exercise and are improved by rest. Which of the following is the most appropriate treatment?
Daily high-dose inhaled corticosteroid and β-agonist when needed
Daily high-dose inhaled corticosteroid with oral steroids for exacerbations and short-acting β- agonist when needed
Daily low-dose inhaled corticosteroid and short-acting β-agonist when needed
Daily oral steroids and long-acting β-agonist
Short-acting β-agonist when needed
187) A 63-year-old woman presents with dyspnea and coughing up foul smelling purulent sputum. She has had many similar episodes in the past. There are no other constitutional symptoms and she denies excessive alcohol intake. On physical examination, she appears chronically ill with clubbing of the fingers. Heart sounds are normal, JVP is measured at 4 cm, and there are inspiratory crackles heard at the lung bases posteriorly. There is no hepatosplenomegaly or any palpable lymph nodes. CXR shows scaring in the left lower lobe, which on chest CT scan is identified as cystic changes with airway dilatation and bronchial wall thickening. Which of the following is the most appropriate initial next step in management?
Antibiotics and postural drainage
Steroids
Radiotherapy
Aerosols
INH
188) A 65-year-old smoker previously diagnosed with chronic obstructive pulmonary disease presents to the emergency department complaining of worsening cough and sputum production. She reports feeling breathless when climbing the stairs to her first floor walk-up apartment, and has moderate difficulty in providing her history in complete sentences. X-ray of the chest shows hyper inflated lungs with flattened diaphragms, attenuated vascular markings, and a narrow mediastinum. What agent(s) will provide the greatest relief of symptoms in the emergency department?
Albuterol and ipratropium bromide
Antibiotics
Magnesium sulfate
N-acetylcysteine
Theophylline
189) A 51-year-old man presenting to the clinic for routine examination mentions that he has not been able to get over the flu. Further questioning clarifies that he has had a nonproductive cough for the past 3–4 months and is unable to sustain his normal walking pace for prolonged periods. He reports feeling more fatigued than he recalls feeling last year. His medical history is significant for hypertension, alcoholism, and obesity. His blood pressure is well controlled on losartan and hydrochlorothiazide. He has no known drug allergies. His mother died of complications of type 2 diabetes mellitus. His father had a fatal myocardial infarction at age 56 years. The patient smokes half a pack of cigarettes per day and has done so for the past 7 years. He has worked all his adult life as an accountant and has no known exposures to asbestos or organic dusts. His blood pressure is 134/96 mm Hg, heart rate is 78/min, respiratory rate is 16/min, temperature is 37°C (98.6°F), and oxygen saturation is 94% on room air. Lung examination reveals bilateral inspiratory crackles in the lung bases. High-resolution CT shows patchy areas of ground glass, reticular abnormalities, and traction bronchiectasis. Laboratory findings are normal except for an elevated erythrocyte sedimentation rate of 54 mm/hr. What is the recommended treatment for this patient’s cough?
Change the antihypertensive agent
Prescribe amantadine
Prescribe bronchodilators and long-term domiciliary oxygen therapy
Prescribe glucocorticoids
Resect diseased lung fields
190) A 38-year-old woman has been complaining of a 2-year history of increasing dyspnea and fatigue. Physical examination reveals increased JVP and a reduced carotid pulse. Precordial examination reveals a left parasternal lift, loud P2, and right-sided S3 and S4. There are no audible murmurs. CXR reveals clear lung fields and an ECG shows evidence of right ventricular hypertrophy. Pulmonary function tests show a slight restrictive pattern. Primary pulmonary hypertension is diagnosed. Which of the following treatments is helpful in this condition?
Corticosteroids
Nitrates
Alpha-adrenergic blockers
Calcium channel blockers
Angiotensin-converting enzyme (ACE) inhibitors
191) A 1-year-old child with cerebral palsy secondary to perinatal asphyxia presents to her general pediatrician for a well-child visit. She was delivered at 37 weeks’ gestation by emergency cesarean section for a tight nuchal chord. The patient has severe spastic quadriparesis that is limiting her movements. She also has mental retardation and is unable to speak. She has received physical and occupational therapy since early infancy; however, her parents are concerned by her lack of improvement. Which of the following is the best choice for treatment of spasticity in this child?
Baclofen
Botulinum toxin
Carbamazepine
Discontinue physical therapy
Hyperbaric oxygen
192) A 66-year-old man has progressive shortness of breath due to COPD. He is currently able to do his activities of daily living, but has trouble walking more than one block. His physical examination reveals hyperinflation, increased resonance to percussion, and bilateral expiratory wheezes. He is on appropriate medical therapy for his stage of COPD. Which of the following is also indicated in the management of this condition?
Meningococcal vaccination
Yearly influenza vaccination
Weight reduction if obese
Haemophilus influenzae B vaccination
Pneumococcal vaccination
193) A 52-year-old man develops sudden-onset shortness of breath on postoperative day 4 after a hemicolectomy for colon cancer. His surgery went well with no operative complications. He reports no cough, sputum, or pleuritic chest pain. His blood pressure is 155/90 mm Hg, pulse 100/min, temperature 37.8°C, lungs are clear, and heart sounds normal. He has bilateral pedal edema, but no discomfort in his legs. His CXR and ECG are normal. A chest CT with contrast reveals a thrombus in his right upper lobe artery. Which of the following statements concerning the management of this condition is most likely correct?
Continuous IV heparin or subcutaneous low-molecular-weight heparin (LMWH) therapy is indicated
Urgent thrombolytic therapy is indicated
Urgent inferior vena cava (IVC)
Filter insertion is indicated
Confirmation of the diagnosis with bilateral ultrasound leg Dopplers
194) A 23-year-old man is seen in the emergency department for sudden onset, right-sided pleuritic chest pain that developed 30 minutes ago while he was watching television. The patient also complains of difficulty breathing. He has no prior medical history, denies smoking and intravenous drug use, and does not take any medications. His temperature is 37.3°C (99.1°F), blood pressure is 130/82 mm Hg, pulse is 92/ min and regular, respiratory rate is 20/min and shallow, and oxygen saturation is 98% on room air. His body mass index is 18 kg/m². Diminished breath sounds, hyperresonance, and decreased tactile fremitus are prominent in the right lung field. The trachea is midline. X-ray of the chest shows a 10% pneumothorax on the right. Which of the following is the most appropriate initial management?
Needle decompression
Observation with supplemental oxygen
Open thoracotomy with oversewing of the pleural blebs and scarification of the pleura
Thoracoscopy with stapling of blebs
Tube thoracostomy with doxycycline pleurodesis
195) A 57-year-old Caucasian man presents complaining of "feeling overwhelmed by life." He has felt depressed for the past six months, and states that he has difficulty focusing on daily activities. His sleep is poor and he wakes frequently at night. Further inquiry reveals that he is a Vietnam War veteran and that he experiences flashbacks and nightmares about his combat experiences. He notes that the flashbacks began prior to his feelings of depression. He avoids watching war movies and refuses to discuss with anyone his time spent in Vietnam. This is his first visit to a psychiatrist and he asks to be started on some medication. Which of the following should be avoided in this patient?
. Lorazepam
. Nortriptyline
. Paroxetine
. Exposure therapy
. Cognitive therapy
196) A 58-year-old woman has a history of major depressive disorder that has been in remission for the past ten years. She takes phenelzine daily and sees her psychiatrist every other month. While on vacation, the patient and her husband indulge in a gourmet meal. Afterward, the patient becomes concerned as she believes one of the dishes may have contained aged cheese, which her psychiatrist had warned her about eating. She comes to the emergency department of the local hospital to express her concerns. Which of the following is most appropriate to monitor?
. Blood pressure
. Blood urea nitrogen and creatinine
. Creatine phosphokinase
. Liver function tests
. Temperature
197) While working as the attending pediatrician in the neonatai ICU, you are called by nursing to examine a two-day-old male child who has recently begun vomiting. The child has failed to pass meconium since birth. Physical examination demonstrates significant abdominal distention. A rectal examination reveals no stool in the vault. You strongly suspect Hirschsprung disease and inform the child's mother that an abdominal radiograph and barium enema are necessary. The mother forbids you from proceeding with any further testing or procedures because she does not believe "there is anything wrong." What is the most appropriate next step?
. Agree not to perform the tests
. Obtain a court order to authorize testing
. Proceed with tests
. Contact the hospital's ethics committee for guidance
. Wait until the mother provides consent
198) A 24-year-old female veterinary assistant is referred to a psychiatrist for the presumptive diagnosis of dysthymia. She reports having a three-year history of low energy levels and gradual worsening in her ability to focus on her work. She feels "sad, hopeless," and experiences little pleasure. She denies any suicidal thoughts. She has never been on any psychotropic medications before, and denies any family history of psychiatric illness. She has gastroesophageal reflux disease, for which she takes omeprazole. She has no known allergies, and does not drink alcohol or smoke cigarettes. The psychiatrist decides to treat her dysthymia with bupropion. Which of the following is a contraindication to the use of bupropion?
. Narcolepsy
. Attention deficit hyperactivity disorder
. Smoking cessation
. Anorexia nervosa
. Bipolar disorder
199) A 35-year-old woman has chronic auditory hallucinations in which she hears her father's voice. His voice said encouraging things to her in the past, but it has recently been scolding her and saying derogatory things about her. The woman is started on risperidone, and she returns two weeks later for a follow-up visit. Although she states that the hallucinations are much better, you notice that she is walking much slower than usual and is not swinging her arms. Also, the patient's facial expressions are rather flat and unchanging, and she admits that her writing has become smaller since starting the medication. Which of the following pharmacologic changes would be most appropriate?
. Start benztropine
. Start dantrolene
. Start propranolol
. Start sertraline
. Discontinue risperidone and start haloperidol
200) A 29-year-old gravida 1, para 0 Caucasian female with a history of bipolar disorder presents at eight weeks of gestation for prenatal counseling. She has been taking a stable dose of lithium for the past year. Her last depressive episode was 16 months ago. Her family history is significant for bipolar disorder in her mother, which was successfully controlled with lithium. She does not smoke cigarettes or consume alcohol. She asks whether her baby will have any congenital abnormalities. Which of the following complications is her fetus at increased risk for?
. Cardiac anomalies
. Craniofacial defects
. Neural tube defects
. Mental retardation
. Genital anomalies
201) A 22-year-old woman is brought to the emergency department by her father because she is demonstrating "strange, disorganized behavior." Earlier that day, she had insisted that the television news anchorman was talking directly to her about the risks of "poison rays" from the moon. She was also apparently attempting to re-organize her bedroom closet when her father found her mumbling incoherently and counting the same pair of socks over and over. She is agitated when examined in the emergency department and the decision is made to admit her to the psychiatric ward. There, after a detailed interview, the diagnosis of schizophrenia is made. She is stabilized with antipsychotics and then discharged home. Her father asks what can be done to ease her return to everyday life and to decrease the likelihood of re-hospitalization. Which of the following measures would be most helpful in this regard?
. Restrict the patient to home as much as possible
. Encourage the patient to return to work
. Minimize the patient's social interaction with others
. Keep family stresses and conflicts to a minimum
. Strongly encourage the patient to make new friends
202) A middle-aged Asian American married couple comes to clinic for a routine examination. Both the husband and wife have been known patients of yours for many years. The husband is being treated for coronary artery disease and tension headaches. Today, he mentions that he "cannot get an erection" even when he feels sexual desire. He adds that he still finds his wife physically attractive and that they are mutually compatible in every respect. This recent development has been causing him significant anxiety and has been a source of concern for his wife. After ruling out medical and psychological causes for his erectile dysfunction by taking a detailed history and pursuing a thorough workup, you agree to his request that he be started on sildenafil. One week later, his wife places an urgent call to your office, reporting that her husband developed severe chest pain "after taking the new medication you prescribed." An EKG done in the emergency department reveals new inferior wall ischemia. You immediately realize that you should not have written him the prescription for sildenafil because he was already using a nitroglycerin patch for his coronary artery disease. What is the most appropriate next step?
. Withhold this information as it could result in a lawsuit
. Admit to the patient and his wife that you made a mistake
. Tell the patient and his wife that you were not aware of this cross-reaction between the two drugs
. Tell the patient and his wife that his chest pain was probably precipitated by anxiety
. Remove yourself as the patient's physician immediately
203) A 38-year-old male with a chronic history of schizophrenia is admitted to the hospital for deterioration in his condition. He is a known patient and was stable on risperidone for the past few years. Today, the patient does not respond when spoken to and he sits motionlessly. He makes no eye contact and his face remains expressionless. Vital signs include temperature of 37.0°C (98.6°F), blood pressure of 132/80 mm Hg, pulse of 88/min, and respirations of 14/min. Physical examination demonstrates diffuse muscle rigidity but is otherwise unremarkable. Which of the following would be the most appropriate next step?
. Increase risperidone dose
. Initiate therapy with dantrolene sodium
. Replace risperidone with quetiapine
. Continue same dose of risperidone and add clozapine
. Administer lorazepam
204) A 7-year-old boy is brought to the pediatrician by his mother for a routine check-up. Physical examination is normal, but during the exam the mother reveals that her son's behavior has been poor. She says that he is always running around (even during dinner), doesn't listen, and keeps his room a mess. She hopes he will grow out of this soon. Upon further questioning, the pediatrician learns that the patient's second-grade teacher has reported that he answers questions impulsively and out of turn, and that he cannot sit at his desk for a very long time without fidgeting. He also frequently forgets to bring in his homework. The mother states that her son has had these symptoms for about a year, and that his grades have been falling over the past six months. The diagnosis of attention deficit hyperactivity disorder is made and the boy is started on methylphenidate. Which of the following side effects is most likely to occur in patients who take methylphenidate?
. Decreased appetite
. Diarrhea
. Hypersomnia
. Seizures
. Weight gain
205) A 16-year-old boy presents to his pediatrician with scattered petechiae and a three-week history of fatigue. Laboratory evaluation and a bone marrow biopsy confirm the diagnosis of acute lymphoblastic leukemia. The boy's parents are divorced and have joint custody of him. His mother consents to the proposed chemotherapy and blood transfusion and requests that his treatment be started immediately. The father, however, refuses to agree to the treatment because of his religious beliefs. What is the most appropriate means of handling this situation?
. Withhold the proposed treatment
. Proceed with the proposed treatment
. Ask the patient his wishes
. Obtain a court order to proceed
. Consult the hospital's ethics committee
206) A middle-aged married couple brings their 10-year-old daughter to the pediatrician because "she just is too tired to play nowadays and sleeps all the time." The parents say that their daughter is winded with minimal exertion. Further inquiry reveals that for the past two weeks, the girl has had bleeding gums when she brushes her teeth. Physical examination demonstrates pallor and hepatosplenomegaly. An extensive workup indicates that the girl is suffering from acute lymphoblastic leukemia, and her parents are informed that chemotherapy will be necessary. The parents firmly refuse to comply with this treatment plan, however, because they are convinced that the side effects will be too severe. Despite a lengthy discussion about the benefits of treatment and the likely consequences for their daughter if therapy is withheld, the girl’s parents insist upon taking her home. What is the most appropriate next step?
. Proceed with the chemotherapy
. Comply with parent’s wishes
. Obtain a court order for chemotherapy
. Provide supportive treatment only
. Inform the parents that if they refuse treatment for their daughter, they will need to find a new pediatrician to assume her care
207) A 72-year-old man presents for a follow-up visit regarding his depressed mood after his wife's death seven months ago. The patient complains of lethargy and episodes of tearfulness. He has attended three sessions of supportive psychotherapy so far and expresses appreciation for the referral because he likes the therapist. However, he continues to feel depressed and is now experiencing insomnia. He is also feeling very guilty about his wife's death and feels that he should have somehow done more for her in her final months. He admits that during a particularly dark moment last month he briefly contemplated committing suicide so that he could "join her," although he is adamant that he does not feel suicidal at this time and will readily contract for safety. Physical examination reveals the patient has lost 13 lbs (5.9 kg) since his wife's death. What is the most appropriate next step?
. Prescribe amitriptyline and alprazolam
. Prescribe sertraline
. Increase psychotherapy sessions to twice weekly
. Recommend electroconvulsive therapy
. Provide reassurance and continue psychotherapy regimen
208) A 60-year-old Caucasian man presents to his family physician complaining of weight loss and fatigue. A review of systems is positive for a change in bowel habits. He says that in the past several weeks, he has developed pencil-thin stools. Further evaluation reveals the presence of colon carcinoma. The patient is told the diagnosis, and his treatment options and prognosis are discussed. He refuses any treatment and says he wants to die peacefully at home without any medical intervention. What would be the most appropriate means of handling the situation?
. Treat the patient against his wishes
. Contact his family and ask them to convince him to receive treatment
. Ask the patient why he is reluctant to be treated
. Respect the patient's decision
. Provide the patient with some literature about the benefits of chemotherapy
209) A 20-year-old female college student is brought to the emergency department by police at 2:30 am after she was caught attempting to enter the White House. She is highly agitated and is pacing around the examination room. Upon further questioning, she states that she just flew in from Texas to meet with the president because she has a "foolproof plan for eliminating terrorism worldwide in just seven days." She adds that she has barely slept for the past ten nights because she has been working intensively on this plan. She speaks very quickly, but periodically stops to angrily shout, "I'm going to sue all of you for interfering with my right to meet with the president." The evaluation has to be stopped because she demands that she be allowed to leave. Family history is significant for major depression in her mother. Vital signs are temperature 37.0°C (98.6°F), blood pressure 148/84 mmHg, pulse 98/min, and respiratory rate 22/min. Which of the following medications should be administered to this patient immediately?
. Mirtazapine
. Desipramine
. Fluoxetine
. Haloperidol
. Lithium carbonate
210) A 52-year-old obese male presents with complaints of episodes of daytime sleepiness that prove irresistible. He reports feeling very refreshed and energized upon awakening from these "naps." A detailed history reveals he also frequently experiences a complete loss of muscle tone, especially when he laughs suddenly or feels a surge of intense emotion. Of particular concern to this patient is that he occasionally finds himself unable to move upon waking, and says he feels temporarily "frozen." His previous diagnoses include gout and external hemorrhoids. His family history is noncontributory. He does not smoke cigarettes or drink alcohol and denies recreational drug use. Physical examination reveals no abnormalities. Which of the following is the most appropriate treatment for this patient?
. Benzodiazepines
. Melatonin
. Treatment of his underlying neurological condition
. Methylphenidate
. Weight reduction
211) A 28-year-old woman is brought to the emergency department by her sister after she experiences a generalized tonic-clonic seizure. The patient appears confused and is unable to answer any questions. The sister says the patient has never had a seizure before. The patient's medical history is unremarkable except for a long history of panic disorder for which she has been taking high doses of "some medication." The sister adds that the patient missed a few doses recently because they were staying at a friend's house, but she is sure that the patient was otherwise regular in taking her medication. Which of the following medications was the patient most likely taking?
. Clonazepam
. Clomipramine
. Buspirone
. Alprazolam
. Paroxetine
212) A 27-year-old man is brought to his primary care physician by his wife because he has not been himself for the past ten days. His wife says that her formerly reserved, quiet husband has been staying up all night lately gambling their savings away while playing poker with friends that he recently met. He is also not concerned about having missed work for the past week. When asked about this recent change in behavior, the man smiles and replies proudly that he no longer needs to work because he has invested a large sum of money in a company that will soon make him rich. He speaks very quickly and it is difficult for him to focus his attention at times. Which one of the following medications would be most appropriate in the treatment of this patient?
. Bupropion
. Clomipramine
. Clozapine
. Haloperidol
. Valproic acid
213) A 27-year-old man presents to his primary care physician complaining of feelings of inadequacy and hopelessness. In a quavering voice, he reports that he gets "extremely anxious" when he has to speak before a group of people at business meetings or other social events. He is unable to "just relax," even at small parties with friends or family, and he isolates himself as a result. He feels his social seclusion is limiting his career advancement, which bothers him greatly. Which of the following treatment combinations would be the most appropriate choice for this patient?
. Fluoxetine and chlordiazepoxide
. Assertiveness training and paroxetine
. Psychodynamic psychotherapy and clonazepam
. Supportive psychotherapy and phenelzine
. Supportive psychotherapy and propranolol
214) A 33-year-old businessman with an unremarkable past medical history presents to you complaining of fatigue and some weight loss over the past three months. He awakens at 4:00 am each morning and is unable to fall back asleep. He also mentions that he used to enjoy playing golf twice per week, but now has little interest. Although he used to excel at work, he now finds himself losing interest and "spacing out" at times. He denies considering suicide. His TSH is normal. He is diagnosed with major depression and prescribed fluoxetine, and is asked to return for a follow-up appointment in two weeks. At the return visit, he states that his symptoms have not worsened, but have not improved either. He says, "I don't feel like the medication is doing anything." What is the most appropriate next step in the management of this patient's condition?
. Continue with fluoxetine at the same dose
. Increase the dose of fluoxetine
. Discontinue fluoxetine and begin sertraline
. Discontinue fluoxetine and begin amitriptyline
. Continue fluoxetine and add amitriptyline
215) A 22-year-old woman presents to the emergency department in an agitated state, certain that she is "about to die." Fifteen minutes ago, she developed heart palpitations and a severe "viselike" tightness in her chest. She experienced a similar episode last week, which resolved in twenty minutes. She denies using any drugs and her medical history is unremarkable. Her father has a history of heart disease and her mother has diabetes mellitus. She takes no medications. Vital signs are temperature 37.0°C (98.6°F), blood pressure 132/74 mmHg, pulse 118/min, and respiratory rate 30/min. She is sweating profusely. EKG reveals sinus tachycardia. The most appropriate next step is administration of which of the following?
. Alprazolam
. Aspirin
. Buspirone
. Fluoxetine
.lmipramine
216) As the on-call surgery resident, you are paged to see a patient on the ward who is having seizures. The patient is a 35-year-old male who underwent a cholecystectomy yesterday. His surgery was uneventful and the man appeared to be doing well until an hour ago when he developed altered sensorium. Upon arriving on the floor, you discover the patient to be confused and agitated. His vital signs include a temperature of 38°C (100.4°F), blood pressure of 134/86 mm Hg, pulse of 110/min, and respirations of 18/min. The man is diaphoretic and has tremors. His wound incisions show no evidence of inflammation, and biliary drainage appears adequate. A review of his medical record indicates his medical history to be unremarkable except for mild esophagitis. He smokes a pack of cigarettes per day and drinks 6-10 beers per night. His medications include folic acid, thiamine, and omepra zole. Laboratory results include the following: CBC: Hb 14.1 g/dL, Leukocyte count 7,100/cmm. Serum Chemistry: Sodium 137 mEq/L, Potassium 4.0 mEq/L, Chloride 101 mEq/L, Bicarbonates 24 mEq/L, BUN 12 mg/dL, Creatinine 1.0 mg/dL, Glucose 104 mg/dL. Arterial Blood Gas: PH 7.40, PO2 94 mm Hg, PCO2 40 mm Hg. Which of the following should be initiated immediately in this patient?
. Disulfiram
. Chlordiazepoxide
. Phenobarbitone
. Flumazenil
. Flumazenil
217) A 35-year-old Caucasian male with a chronic history of paranoid schizophrenia presents to the mental health clinic. He strongly believes that his wife recently stole his pill bottle of risperidone, which he feels caused him to start experiencing more frequent and intense auditory hallucinations. He says that the "voices" tell him to kill his wife because she cannot be trusted. He admits to having homicidal thoughts about his wife but denies any specific plan for harming her. He requests a refill of his risperidone. What is the most appropriate next step?
. Refill his prescription of risperidone and call his wife after he leaves
. Refill his prescription of risperidone without calling his wife as he does not have a plan to hurt her
. Admit the patient to the psychiatric ward
. Call the patient's wife immediately before filling the risperidone prescription
. Increase his dose of risperidone
218) A 37-year-old obese man returns to his doctor for a follow-up visit. One month ago, he was diagnosed with type II diabetes mellitus and was started on metformin. His fasting glucose is 122 mg/dl, and his blood pressure is 145/85 mm Hg. The patient looks rather dejected and admits it has been difficult to come to terms with his diagnosis. He has tried to exercise, watch his diet, and quit smoking as was suggested, but lately he has been feeling unmotivated and without energy. He admits to feeling sad and guilty about his weight, but is not sure he will be able to do anything about it. He continues to smoke despite attempts to cut back, and has been spending much of the day in bed watching television or sleeping. He has withdrawn from friends and family and took the last two weeks off from work, as he did not feel "up to going." Which of the following would be the best medication to prescribe at this time?
. Bupropion
. Fluoxetine
. Modafinil
. Selegiline
. Venlafaxine
219) A 28-year-old attorney presents complaining of a six-month history of extreme nervousness, irritability, and restlessness. He is "plagued" by recurrent thoughts of not being able to properly carry out his responsibilities at the office. He dreads the possibility of not being offered a position as partner in the law firm. As a result, he finds himself unable to concentrate well or perform his duties efficiently. He is also engaged to be married, and spends sleepless nights thinking about the responsibilities of married life. He worries that he will not be a good husband or father. He denies any history of alcohol intake, but has recently begun drinking two bottles of beer every night for the past month as a means of escaping from his distressing thoughts. Which of the following is the treatment of choice for this patient?
. Propanolol before a distressing situation
. Alprazolam
. Treatment for alcohol dependence
. Ziprasidone
. Buspirone
220) A 16-year-old female presents to her pediatrician's office for a routine examination. She is wearing a sweater despite the warm weather, and admits she often feels cold. She states that she has otherwise been feeling fine, except for having difficulty losing weight. She wants to lose about 10 lbs (4.5 kg) because she believes she is "too fat." She has been on a strict diet and exercises two hours daily. She is also requesting a pregnancy test because she has not had a menstrual period for the past three months. Past medical history is significant for a supracondylar fracture of her right humerus at the age of seven. She is alert, cooperative, and in no apparent distress. Vital signs are temperature 36.2°C (97.2°F), blood pressure 88/58 mmHg, pulse 56/min, and respiratory rate 16/min. Her height is 5'5" (165 cm) and weight is 90 lbs (40.9 kg). What is the most appropriate next step in the management of her symptoms?
. Advise the patient that her complaints likely stem from a thyroid disorder
. Advise the patient that she is underweight and needs to increase her caloric intake
. Refer the patient for cognitive-behavioral therapy to address her weight
. Recommend the patient to begin taking sertraline
. Hospitalize the patient
221) A 21-year-old male is admitted to the intensive care unit after attempting to commit suicide by overdosing on some pills he found in the medicine cabinet at home. A psychiatry consult is ordered. While interviewing the patient, the psychiatrist observes that the patient has a "blank" expression on his face and is minimally responsive. He refuses to make eye contact and speaks in monosyllables. An attempt is made to obtain a more detailed history from the patient's father. He describes his son as very aloof, having avoided the other children in the neighborhood and participated in few activities as a child. His son dropped out of school in ninth grade and has not been able to maintain jobs throughout the years because of his social difficulties. He adds that his son has been increasingly isolated this past year and that he has complained his deceased mother frequently asks him to "join her in heaven." Which of the following medications would provide the greatest benefit to this patient?
. Fluphenazine injections
. Risperidone
. Haloperidol
. Thioridazine
. Chlorpromazine
222) A 26-year-old graduate student presents at her husband's urging, complaining of severe pain during sexual intercourse. She says that she was a virgin when she married her husband two years ago, and that she has been experiencing severe "genital pain" during sex since then. As a result, she avoids sexual intimacy with her husband, which is placing a strain upon their marriage. She also complains of intense pain with her menses and when passing stool. She admits to sporadic pelvic pain that waxes and wanes with no discernible trigger. What would be the most appropriate treatment given this woman's condition?
. Use of vaginal dilators
. Pain management training
. Oral contraceptive pills
. Regularly scheduled follow-up visits
. Psychotherapy and sexual education
223) A 28-year-old male presents for a follow-up visit regarding his bipolar disorder, which has been managed with valproic acid for several years. He is accompanied by his mother who is very concerned because her son has been behaving strangely for the past two weeks, saying that he can hear the voice of his long-deceased father telling him to "come to me." The patient insists that he is able to ignore his father's voice and that he is not considering suicide at this time. The decision is made to start risperidone to treat the auditory hallucinations. This medication predominantly produces its antipsychotic effect by blocking which of the following receptors?
. Alpha-1 adrenergic receptors
. Dopamine-D2 receptors
. Histamine- 1 receptors
. Muscarinic cholinergic receptors
. Serotonin 2A receptors
224) A 59-year-old man well known to his primary care physician presents complaining of excessive fatigue during the daytime. He is accompanied by his wife, who says that he wakes frequently during the night and snores loudly. She worries because sometimes it sounds like he is gasping for breath at night. The only finding on physical examination is morbid obesity. The diagnosis of obstructive sleep apnea is made. Which of the following is not true about this disorder?
. Hypertension is a frequent complication
. Weight loss may improve symptoms
. Apneic episodes may increase with age
. Cor pulmonale can develop as a complication
. Benzodiazepines are effective treatment
225) A 19-year-old college student presents for a first visit because she is concerned about her "abnormal behavior." She says that she worries excessively about her door being locked and that she wakes several times throughout the night to confirm that the door is properly locked. She finds these thoughts distressing. Although she agrees that her fears are not rational, she feels unable to stop from acting on them. This frequent waking grossly impairs her sleep and as a result, she is finding it increasingly difficult to perform well academically. She has no history of psychiatric conditions, but her mother was recently diagnosed with trichotillomania. Vital signs include a temperature of 37.0°C (98.6°F), blood pressure of 118/84rnrnHg, pulse of 76/min, and respirations of 12/min. Physical examination is unremarkable. Which of the following medications would be the most appropriate treatment for her condition?
. Buspirone
. Clomipramine
. Olanzapine
. Paroxetine
. Propanolol
226) A 68-year-old woman with a history of well-managed sick sinus syndrome and osteoporosis is brought to the physician by her daughter. According to the daughter, her mother has become increasingly forgetful and irritable over the past two years. She adds that she felt less concerned about her mother's forgetfulness as "it just comes with aging," but that the "mood swings have become a problem in terms of her getting along with family members." A Folstein Mini-Mental Status Examination is performed and suggests significant cognitive decline. Dementia is strongly suspected. After potentially causative medical or psychiatric conditions have been ruled out, the diagnosis of Alzheimer's dementia is made. Which of the following medications would be most appropriate in the treatment of this woman's condition?
. Sertraline
. Risperidone
. Donepezil
. Lorazepam
. Bupropion
227) A 34-year-old male calls his primary care physician's office, requesting an urgent appointment to discuss a personal problem. The receptionist informs him that the physician is preparing to leave as it is near closing time, but that an appointment is available for early the next morning. The patient declines the offer of an appointment and hangs up. Fifteen minutes later, as the physician is locking the office up, the patient arrives and insists on being seen immediately. He breathlessly says that he has "some kind of red rash" on his penis, which has him very worried. He asks again to be examined now. What is the most appropriate response?
."It is extremely inappropriate on your part to come here despite my request that you make an appointment."
. "All right, we should look at your rash now. Come inside."
. "Your rash can most certainly wait until tomorrow."
. "Didn't I already inform you that it is closing time?''
. 'Although I understand your concern, we should address the issue tomorrow since it is not an emergency."
228) A 19-year-old known patient with bipolar disorder presents to her psychiatrist for a follow-up visit. She experienced her first manic episode three years ago and was then immediately placed on lithium. She has had no relapses and remains asymptomatic. Today she inquires about the possibility of discontinuing her medication. Her mother is apprehensive, however, about the prospect and asks the psychiatrist to continue the medication for a longer period. Which of the following is the most appropriate management of this patient's condition?
. Taper and discontinue lithium
. Continue lithium for six more months in light of mother's concerns
. Order serum lithium levels
. Continue lithium for her lifetime
. Discontinue lithium and initiate valproic acid
229) A fourth-year medical student develops a new onset headache that he describes as "excruciating." He denies any fever or vomiting. He requests the emergency department physician to order a CT scan of his head, as he is certain that he has developed an intracranial hemorrhage. A complete workup, including an ophthalmologic evaluation, is negative. In spite of reassurance from the physician, the student continues to be excessively worried and is unable to focus on his studies due to persistent thoughts about having a "brain bleed." Which of these would be the most effective step in resolving this patient's symptoms?
. Initiate a discussion about current emotional stressors
. Provide gentle reassurance
. Patently explain the benign nature of the headache
. Prescribe a benzodiazepine
. Treat the headache with a placebo
230) A 44-year-old woman presents for a follow-up visit regarding her severe depression. She has taken numerous antidepressants over the years with little symptomatic relief, and continues to spend the majority of her time in seclusion, experiencing intense feelings of worthlessness and despair. Her husband is deeply concerned about the impact her depression has had on their marriage and their two teenage children. He requests information about "all available options" to improve his wife's symptoms. The prospect of electroconvulsive therapy (ECT) is discussed, and the husband inquires about potential drawbacks to the procedure. Which of the following is a common side effect of electroconvulsive therapy?
. Amnesia
. Elevated liver enzymes
. Epilepsy
. Muscle paralysis
. Neuroleptic malignant syndrome
231) A 42-year-old Caucasian man presents to the emergency department complaining of a two-day history of fever to 40°C (104°F), headache, and vomiting. Physical examination reveals petechiae and purpura on his trunk and lower extremities. Examination of his cerebrospinal fluid is consistent with bacterial meningitis. The patient is told of his diagnosis and the need for antibiotics and hospitalization is discussed. The patient refuses to be admitted to the hospital, however, and insists upon being treated at home. What is the most appropriate next step?
. Treat the patient against his wishes
. Discuss the situation with the patient's wife and ask that she convince him to accept hospitalization
. Consult with the hospital ethics committee
. Respect the patient's decision and arrange for home antibiotic therapy
. Obtain a court order to proceed with treatment
232) A 28-year-old married nurse who works with you in clinic comes to you privately, saying that she "missed a period" this month. An over-the-counter urine pregnancy test confirms that she is pregnant. The first-trimester laboratory testing is performed, and she is found also to be HIV-positive. When you reveal the news of her HIV status to her, she is devastated. She hesitantly reveals that she had unprotected sexual intercourse with a former boyfriend several months ago. You explain that her husband's HIV status must be evaluated. Upon hearing this, the nurse is horrified and says. "No, I cannot tell him about this. He would never forgive me!" What is the most appropriate next step?
. Call and inform local health authorities immediately
. Call and inform local health authorities and the nurse's husband immediately
. Assure the nurse that her condition will be kept absolutely confidential
. Encourage the nurse to inform her husband
. Tell the nurse that if she insists on hiding her HIV status from her husband, she cannot expect your medical or moral support
233) You are seeing a 41-year-old woman and her 19-year-old daughter in the emergency room because they think that they have been poisoned. The woman states that they have not been getting along with their landlord lately because they have been late on their last few rent payments. They say that the landlord is harassing them by banging on their floor at all hours of the night, turning down their heat, and asking the other tenants to spy on them. Last month, the woman thinks the landlord tried to spy on them by installing video cameras in her living room while they were out. Today, she tasted something odd in her food, and is convinced that she and her daughter have been poisoned in an attempt to get them out of the apartment once and for all. She came immediately to the hospital for treatment and to collect evidence of the poisoning. The mother is very protective of her daughter, who seems rather submissive and passive. She completely agrees with her mother's description of the events. Which of the following is the best course of action?
. Discharge both the mother and the daughter after a complete physical examination and blood tests
. Admit the mother to the psychiatric unit for treatment, but not the daughter
. Admit the daughter to the psychiatric unit for treatment, but not the mother
. Admit the mother and the daughter to the same psychiatric unit
. Admit the mother and the daughter to different psychiatric units
234) A 24-year-old man with chronic schizophrenia is brought to the emergency room after his parents found him in his bed and were unable to communicate with him. On examination, the man is confused and disoriented. He has severe muscle rigidity and a temperature of 39.4°C (103°F). His blood pressure is elevated, and he has a leucocytosis. Which of the following is the best first step in the pharmacologic treatment of this man?
. Haloperidol
. Lorazepam
. Bromocriptine
. Benztropine
. Lithium
235) A 30-year-old man is brought to the emergency room after he was found wandering on the streets with no shoes on in the middle of winter. He is admitted to the inpatient psychiatric unit and stabilized on antipsychotic medication. Looking at past records, his psychiatrist notes that he is repeatedly noncompliant with his medication postdischarge, and each time he relapses within 6 months. Which of the following medications is the best one for this patient to be maintained on?
. Clozapine
. Haloperidol decanoate
. Chlorpromazine
. Thioridazine
. Quetiapine
236) A 23-year-old woman was diagnosed with schizophrenia after a single episode of psychosis (hallucinations and delusions) that lasted 7 months. She was started on a small dose of olanzapine at the time of diagnosis, which resulted in the disappearance of all her psychotic symptoms. She has now been symptom free for the past 3 years. Which of the following treatment changes should be made first?
. Her olanzapine should be decreased and then stopped if she remains symptom free
. Her olanzapine should be decreased, but not stopped
. Her olanzapine should be maintained at a constant level, but she can stretch out the time between her appointments with the psychiatrist
. Her diagnosis should be reexamined as she is likely not schizophrenic at all
. Her olanzapine should be switched to a long-acting depot antipsychotic medication such as haloperidol decanoate
237) A 28-year-old woman is diagnosed with bipolar disorder, manic type, when she was hospitalized after becoming psychotic, hypersexual, severely agitated, and unable to sleep. She is started on a medication in the acute phase of her illness. Which of the following medications, recommended for acute use in manic patients, is recommended to be continued on into maintenance therapy?
. Aripiprazole
. Lamotrigine
. Lithium
. Olanzepine
. Ziprasidone
238) A 30-year-old man comes to the psychiatrist for the evaluation of a depressed mood. He states that at least since his mid-20s he has felt depressed. He notes poor self-esteem and low energy, and feels hopeless about his situation, though he denies suicidal ideation. He states he does not use drugs or alcohol, and has no medical problems. His last physical examination by his physician 1 month ago was entirely normal. Which of the following treatment options should be tried first?
. ECT
. Hospitalization
. Psychoanalysis
. Venlafaxine
. Amoxapine
239) A 22-year-old college student calls his psychiatrist because for the past week, after cramming hard for finals, his thoughts have been racing and he is irritable. The psychiatrist notes that the patient’s speech is pressured as well. The patient has been stable for the past 6 months on 500 mg of valproate twice a day. Which of the following is the most appropriate first step in the management of this patient’s symptoms?
. Hospitalize the patient
. Increase the valproate by 500 mg/day
. Prescribe clonazepam 1 mg qhs
. Start haloperidol 5 mg qd
. Tell the patient to begin psychotherapy one time per week
240) A 38-year-old woman with bipolar disorder has been stable on lithium for the past 2 years. She comes to her psychiatrist’s office in tears after a 2-week history of a depressed mood, poor concentration, loss of appetite, and passive suicidal ideation. Which of the following is the most appropriate next step in the management of this patient?
. Start the patient on a second mood stabilizer
. Start the patient on a long-acting benzodiazepine
. Stop the lithium and start an antidepressant
. Start an antidepressant and continue the lithium
. Stop the lithium and start an antipsychotic
241) A 42-year-old woman sees her physician because she has been depressed for the past 4 months. She also notes that she has gained 20 lb without trying to. She notes that she does not take pleasure in the activities that she once enjoyed and seems fatigued most of the time. These symptoms have caused the patient to withdraw from many of the social functions that she once enjoyed. The physician diagnoses the patient with hypothyroidism and starts her on thyroid supplementation. Six weeks later, the patient’s thyroid hormone levels have normalized, but she still reports feeling depressed. Which of the following is the most appropriate next step in the management of this patient?
. Recommend that the patient begin psychotherapy
. Increase the patient’s thyroid supplementation
. Start the patient on an antidepressant medication
. Tell the patient that she should wait another 6 weeks, during which time her mood will improve
. Take a substance abuse history from the patient
242) A 54-year-old man sees a physician complaining of a depressed mood and inability to sleep for the past 3 weeks. He tells the physician that in the past when he has had similar episodes, he was placed on a monoamine oxidase inhibitor, which proved effective. The physician diagnoses the patient with major depression and agrees to use an MAOI. Which of the following foods must be completely avoided by this patient while on this medication?
. Licorice
. Coffee
. Chocolate
. Cheddar cheese
. Soy sauce
243) A 10-year-old boy is brought to the psychiatrist by his mother. She states that for the past 2 months he has been increasingly irritable, withdrawn, and apathetic. He has been refusing to do his homework, and his grades have dropped. Which of the following is the best next step in management?
. The child should be hospitalized
. The child should be started in supportive psychotherapy
. The mother should be warned that the child will likely turn out to be bipolar (67% chance)
. The child should receive an antidepressant medication
. The child should receive lithium and an antidepressant
244) A 35-year-old woman is seeing a psychiatrist for treatment of her major depression. After 4 weeks on fluoxetine at 40 mg/day, her psychiatrist decides to try augmentation. Which of the following is the most appropriate medication?
. Lithium
. Sertraline
. An MAO inhibitor
. Clonazepam
. Haloperidol
245) While working as the medical resident on call, you are paged to see a 72-year-old woman who complains of a headache. Upon seeing you, she snaps, "You look young and foolish. I hope you're more efficient than you seem." While you are performing a basic physical examination on her, she says, "I'm going to complain to the hospital administrators about you because you have no idea how to properly deal with a patient." Which of the following is the best response to her threat?
. "Feel free to do as you please."
. "Please allow me to complete the physical examination first."
. "You seem to be angry, and I don't know why."
. "You're hurting me with your comments."
. "Would you prefer that my attending see you instead?"
246) A colleague who works as a pediatrician in your group practice comes to your office to inquire about a patient you examined earlier this morning. The colleague says that he recognized the patient as a former neighbor of his who is also a good friend of the family. The patient was recently diagnosed as HIV-positive and is now on antiretroviral medication. Your colleague asks what the patient is being treated for. What is the most appropriate response to such an inquiry?
. "He is on anti-retrovirals because he is HIV positive.”
. "Actually, we've not been able to diagnose his condition."
. "It is unprofessional of you to ask me this question."
. "I don't think it would be appropriate for me to discuss this with you."
. "I don't think it is necessary for me to discuss this with you."
247) A 35-year-old female presents to your office complaining of knee pain. She has a long history of rheumatoid arthritis affecting her hand and knee joints. The stiffness and pain has persisted over the last several months and has responded poorly to NSAIDs. Over the last two days the pain in her right knee has been interfering with her sleep. She had an episode of chills this morning. Her blood pressure is 120/70 mmHg, pulse is 90/min, temperature is 38.7°C (102°F) and respiratory rate is 18/min. Physical examination reveals swelling in the joints of her hand and wrist. The right knee is red and swollen; active and passive range of motion at the right knee is limited due to pain. This patient's current condition can be best treated with which of the following?
. Colchicine
. Corticosteroids
. Anti-cytokine agents
. Antibiotics
. Antimetabolites
248) A 68-year-old African-American woman presents to the ER in acute distress. She complains of a sudden onset of complete visual loss in her right eye. Her vision had been blurry for the past few days and acutely worsened one hour ago. She also describes right-sided headaches of two months duration for which she has taken ibuprofen without relief. She has no nausea or vomiting. She has diabetes, hypertension, and degenerative joint disease. Her temperature is 37.2°C (98.9°F) and blood pressure is 146/86 mmHg. Examination reveals complete loss of vision in the right eye. Her pupils are 4 mm bilaterally. Fundoscopy shows a swollen pale disc with blurred margins. A bruit is heard in the right subclavicular area. Motor and sensory examination is within normal limits. Which of the following is the most appropriate next step in management?
. Temporal artery biopsy
. Low dose prednisone
. Methotrexate therapy
. MRI of the brain with contrast
. High dose prednisone
249) A 45-year-old woman presents to your office complaining of one week of fatigue and weakness in her shoulders and hips. She reports difficulty performing household tasks, climbing the stairs, or rising from a chair. She denies any facial weakness or blurry vision. She has a history of gastroesophageal reflux disease and takes over the counter ranitidine. Her family history is positive for breast cancer in her mother. Her vital signs are within normal limits and examination shows normal sensation and deep tendon reflexes. Strength is 4 out of 5 in the proximal muscles groups of the arms and legs, and there is no limitation in the range of motion. Other physical examination is within normal limits. The ESR is 80 mm/hour and her serum creatine kinase is elevated. The TSH is within normal limits. Which of the following is the best initial treatment for this patient?
. Indomethacin
. Corticosteroids
. Colchicine
. Riluzole
. Amitriptyline
250) A 55-year-old man presents to his physician with 6 months of mild intermittent pain in both knees. The pain is aggravated by walking and weight bearing, and is partially relieved by rest. He also complains of morning stiffness that lasts for 10 minutes. He does not smoke but drinks alcohol occasionally. His BMI is 25 Kg/m2, temperature is 37°C (98.6°F), pulse is 75/min, and blood pressure is 130/80 mmHg. Examination reveals crepitus over both knee joints. Which of the following is the most appropriate initial treatment for this patient?
. Antibiotics
. Ketorolac
. Acetaminophen
. Colchicine
. Oxycodone
251) A 38-year-old woman complains of chronic aching pain and stiffness around the neck, shoulders, low back and hips. She fatigues easily and has been having problems with sleep. Even minor exertion worsens her pain. She has no muscle weakness, fever, malaise, weight loss or rash. She has a history of irritable bowel syndrome. Her vital signs are within normal limits. Examination shows multiple tender spots over specific points on her body. Power is 5/5 in all extremities and deep tendon reflexes are 2+. No sensory abnormalities are noted. Labs show: ESR 9 mm/hr, Hematocrit 43%, WBC count 7,000/microL, Platelet count 200,000/microL, TSH 3 microU/L, CPK 100 IU/L. Which of the following is the most appropriate pharmacotherapy for this patient?
. Naproxen
. Amitriptyline
. Prednisone
. Colchicine
. Oxycodone
252) A 35-year-old man presents to your office with severe back pain. The pain started three days ago when the patient was carrying a heavy pack. The pain radiates to the posterior surface of the right thigh. The straightleg raise test is positive. There is no bladder or bowel incontinence. Pain and temperature perception is preserved in the affected extremity, as well as in the perineal area. Anal reflex is normal. Which of the following is the best next step in the management of this patient?
. MRI of the spine
. CT scan of the spine
. Early mobilization and NSAIDs
. Plain roentgenogram
. Surgical decompression
253) A 31-year-old Caucasian male complains of joint pains. He describes right knee pain, right heel pain and lowback pain. He was recently treated for urethral discharge at an outside clinic. He has no history of trauma or illicit drug use. He is afebrile, and his vital signs are stable. His right knee is swollen, tender and warm to touch; tenderness is also present over the Achilles tendon. Oral examination shows mouth ulcers. Synovial fluid analysis from the right knee shows a white blood cell count of 10,000/mm3 with many polymorphonuclear leukocytes but a negative Gram stain. Which of the following is the most appropriate initial pharmacotherapy for this patient?
. Antihistamines
. NSAIDs
. Antibiotics
. Colchicine
. Allopurinol
254) A 70-year-old male presents with malaise, pain and stiffness of his neck, shoulders and hips for the last 3 months. His stiffness is worse in the morning and lasts about 30-40 minutes. He also complains of a recent weight loss of 7 1bs and mild fevers. He has no headache, scalp tenderness, visual symptoms or jaw claudication. Examination shows no swelling or tenderness of the involved joints with normal range of motion on active and passive movement. The arteries of the scalp, neck and extremities are normally palpable without any tenderness. Labs show a hematocrit of 31% and an ESR of 65 mm/hr. Blood cultures are negative. Which of the following is the most appropriate next step in management of this patient?
. Temporal artery biopsy
. Treatment with nonsteroidal anti-inflammatory agents
. Treatment with low-dose prednisone
. Treatment with high-dose prednisone
. Measure antinuclear antibodies and rheumatoid factor levels
255) A 43-year-old construction worker presents to your office 2 weeks after an episode of acute back pain. The pain started after lifting a heavy box, was localized to the lower back with little radiation to the buttocks, and quickly responded to conservative treatment. His past medical history is insignificant. He smokes 2 packs a day and consumes alcohol occasionally. He is concerned about the possibility of recurrence of the pain. Which of the following would be most helpful in preventing another episode of back pain in this patient?
. Limiting the physical activity as much as possible
. Sleep on the stomach
. Regular exercise with repetitive twisting and bending
. Bend at the waist, not at the knees
. Keep the back straight while lifting an object
256) A 54-year-old woman comes to the physician's office complaining of chronic, bilateral knee and hip pain. The pain increases with activity and is relieved by rest. She denies fever, chills or weight loss. The review of systems is unremarkable. She currently weighs 80 kg (180 lb) and is 146 cm (59 in) tall. Her knee joints are tender but there is no warmth, erythema or effusion. X-rays show narrow joint space, subchondral bone cysts, and bony spurs in both knees. Which of the following interventions would provide the greatest long-term benefit to this patient?
. Muscle strengthening exercise
. Non-steroidal antiinflammatory agents
. Weight loss
. Chondroitin sulfate
. Acetaminophen with codeine
257) A 45-year-old man complains of sudden pain and swelling in his left first metatarsophalangeal joint. He is undergoing high dose induction chemotherapy for acute leukemia. Joint fluid aspiration reveals negative birefringent crystals and elevated white cell count. Which of the following prophylactic measures would most likely have prevented this condition?
. Urine acidification
. N-acetylcysteine
. Allopurinol
. Mesna
. Adequate hydration
258) A 30-year-old Caucasian female patient is seen at the rheumatology clinic. She has a 4-year history of rheumatoid arthritis. Over the past year, she has noticed an improvement in her symptoms. Examination of her joints reveals less swelling and erythema than on the previous visit Laboratory studies show: CBC: Hb 10.8 g/dL, Ht 32%, MCV 104 fl, Platelet count 226,000/cmm, Leukocyte count 7,500/cmm, Neutrophils 65%, Eosinophils 1%, Lymphocytes 28%, Monocytes 6%. Serum: Serum Na 140 mEq/L, Serum K 3.9 mEq/L, Chloride 100 mEq/L, Bicarbonate 18 mEq/L, BUN 16 mg/dL, Serum Creatinine 1.1 mg/dL, Calcium 9.8 mg/dL, Blood Glucose 98 mg/dL. Which of the following medications is this patient most likely taking?
. Hydroxychloroquine
. Prednisone
. Cyclosporin
. Azathioprine
. Methotrexate
259) A 46-year-old woman suffering from systemic lupus erythematosus (SLE) develops hematuria and proteinuria. She was diagnosed with SLE four years ago and has been treated with hydroxychloroquine. A renal biopsy is performed and shows diffuse proliferative glomerulonephritis. She is started on cyclophosphamide and prednisone and hydroxychloroquine is stopped. She continues these two medications for several weeks because of the low therapeutic response. This new therapeutic regimen puts her at an increased risk for which of the following conditions?
. Cochlear dysfunction
. Optic neuritis
. Bladder carcinoma
. Peripheral neuropathy
. Digital vasospasm
260) A 35-year-old white female presents with pain and stiffness of her wrist and hand joints for the last several months. Her morning stiffness lasts for more than an hour. She also complains of redness and joint swelling. Her past medical history is significant only for a similar episode one year ago. That episode resolved with over the counter ibuprofen. Examination of her joints shows redness, warmth, swelling and tenderness of proximal interphalangeal joints, metacarpophalangeal joints and wrists. X-ray shows periarticular osteopenia and erosions of the proximal interphalangeal and metacarpophalangeal joints. She started taking indomethacin with good relief. Which of the following is the most appropriate next step in the management of this patient?
. Glucocorticoids
. Methotrexate
. Celecoxib
. Azathioprine
. Etanercept
261) A 40-year-old man complains of exquisite pain and tenderness in the left ankle. There is no history of trauma. The patient is taking hydrochlorothiazide for hypertension. On examination, the ankle is very swollen and tender. There are no other physical examination abnormalities. Which of the following is the best next step in management?
. Begin colchicine and broad-spectrum antibiotics.
. Perform arthrocentesis.
. Begin allopurinol if uric acid level is elevated.
. Obtain ankle x-ray to rule out fracture.
. Apply a splint or removable cast.
262) A 48-year-old woman complains of joint pain and morning stiffness for 4 months. Examination reveals swelling of the wrists and MCPs as well as tenderness and joint effusion in both knees. The rheumatoid factor is positive, antibodies to cyclic citrullinated protein are present, and subcutaneous nodules are noted on the extensor surfaces of the forearm. Which of the following statements is correct?
. Prednisone 60 mg per day should be started.
. The patient has RA and should be evaluated for disease-modifying antirheumatic therapy.
. A nonsteroidal anti-inflammatory drug should be added to aspirin.
. The patient’s prognosis is highly favorable.
. The patient should receive a 3-month trial of full-dose nonsteroidal anti-inflammatory agent before determining whether and/or what additional therapy is indicated
263) A 60-year-old man complains of pain in both knees coming on gradually over the past 2 years. The pain is relieved by rest and worsened by movement. The patient is 5ft 9 in. Tall and weighs 210 lb. There is bony enlargement of the knees with mild warmth and small effusions. Crepitation is noted on motion of the knee joint bilaterally. There are no other findings except for bony enlargement at the distal interphalangeal joint. Which of the following is the best way to prevent disease progression?
. Weight reduction
. Calcium supplementation
. Total knee replacement
. Long-term nonsteroidal anti-inflammatory drug (NSAID) administration
. Oral prednisone
264) A 20-year-old woman has developed low-grade fever, a malar rash, and arthralgias of the hands over several months. High titers of anti-DNA antibodies are noted, and complement levels are low. The patient’s white blood cell count is 3000/μL, and platelet count is 90,000/μL. The patient is on no medications and has no signs of active infection. Which of the following statements is correct?
. If glomerulonephritis, severe thrombocytopenia, or hemolytic anemia develops, high-dose glucocorticoid therapy would be indicated.
. Central nervous system symptoms will occur within 10 years.
. The patient can be expected to develop Raynaud phenomenon when exposed to cold.
. Joint deformities will likely occur.
. The disease process described is an absolute contraindication to pregnancy.
265) A 75-year-old man complains of headache. On one occasion he transiently lost vision in his right eye. He also complains of aching in the shoulders and neck. There are no focal neurologic findings. Carotid pulses are normal without bruits. Laboratory data show a mild anemia. Erythrocyte sedimentation rate (ESR) is 85. Which of the following is the best approach to management?
. Begin glucocorticoid therapy and arrange for temporal artery biopsy.
. Schedule temporal artery biopsy and begin corticosteroids based on biopsy results and clinical course.
. Schedule carotid angiography.
. Follow ESR and consider further studies if it remains elevated.
. Start aspirin and defer any invasive studies unless further symptoms develop.
266) A 55-year-old man with psoriasis has been troubled by long-standing destructive arthritis involving the hands, wrists, shoulders, knees, and ankles. Hand films demonstrate pencil-in-cup deformities. He has been treated with naproxen 500 mg bid, sulfasalazine 1 g bid, prednisone 5 mg qd, and methotrexate 17.5 mg once a week without substantive improvement. Which of the following treatments is most likely to provide long-term benefit?
. Cyclophosphamide
. Addition of folic acid supplementation
. Oral cyclosporine
. Tumor necrosis factor alpha inhibitor
. Higher-dose steroids in the range of 20 mg of prednisone per day
267) A 35-year-old woman complains of aching all over. She says she sleeps poorly and all her muscles and joints hurt. Her symptoms have progressed over several years. She reports she is desperate because pain and weakness often cause her to drop things. Physical examination shows multiple points of tenderness over the neck, shoulders, elbows, and wrists. There is no joint swelling or deformity. A complete blood count and erythrocyte sedimentation rate are normal. Rheumatoid factor is negative. Which of the following is the best therapeutic option in this patient?
. Graded aerobic exercise
. Prednisone
. Weekly methotrexate
. Hydroxychloroquine
. A nonsteroidal antiinflammatory drug
{"name":"Ichigo USLME Med Management P3", "url":"https://www.quiz-maker.com/QPREVIEW","txt":"1) A 10-year-old girl with blue eyes and blonde hair is brought to the office by her mother for a routine check-up. All her immunizations are up to date. Her family history is significant for myocardial infarction in her father and schizophrenia in a maternal uncle. There is no family history of any skin malignancies. Her height is at the 60th percentile, and weight is at the 56th percentile. While you are examining her, the mother says with much concern that she saw a television program that claimed that the incidence of skin cancer is increasing dramatically. She wants to know the best way to prevent skin cancer in her daughter, especially since they live in California. What is the best advice to help prevent malignant melanoma in this child?","img":"https://cdn.poll-maker.com/20-807554/mp3-9.png?sz=1200"}
Powered by: Quiz Maker